Download as pdf or txt
Download as pdf or txt
You are on page 1of 53

SAMIKSHA TEST 7 - SOLUTION

Q1. Consider the following differences between the Terrestrial and the Jovian planets:
1. The Terrestrial planets have higher density than the Jovian planets.
2. The Jovian planets have more number of moons than the Terrestrial planets.
3. The Terrestrial planets are smaller than the Jovian planets.
4. The Jovian planets have lower gravity than the Terrestrial planets.
How many of the above statements is/are correct?
A. Only one
B. Only two
C. Only three
D. All four
Answer: C
Explanation:
• Statement 1 is correct: Terrestrial planets (Mercury, Venus, Earth, and Mars) are composed mostly of rock and metal,
making them denser. In contrast, Jovian planets (Jupiter, Saturn, Uranus, and Neptune) are composed mainly of
gases, which results in lower average densities.
• Statement 2 is correct: Jovian planets tend to have more moons compared to Terrestrial planets. As of Feb-2024,
Terrestrial planets have 3 moons (Earth-1, Mars-2), whereas Jovian planets have 285 moons (Jupiter-95, Saturn-146,
Uranus-28, Neptune-16).
• Statement 3 is correct: Terrestrial planets are smaller in terms of both size and mass compared to Jovian planets.
Jovian planets are much larger and more massive due to their composition of gases and liquids. For example, Jupiter
is more than 11 times the diameter of Earth and its volume is over 1,300 times the volume of Earth.
• Statement 4 is not correct: Jovian planets typically have higher gravity than Terrestrial planets because gravity
depends on mass, and Jovian planets have much greater masses. The higher mass of Jovian planets results in stronger
gravitational forces compared to the Terrestrial planets.
Hence, option C is correct.

Q2. Consider the following statements with reference to Earthquake Waves:


1. Body waves are faster than Surface waves and arrive first at the Surface, hence they are more damaging than surface
waves.
2. S-Waves propagation reveals that the outer core of the Earth is in liquid form.
3. P-Waves propagate longitudinally while S-Waves propagate transversally.
How many of the above statements is/are correct?
A. Only one
B. Only two
C. All three
D. None
Answer: B
Explanation:
An earthquake in simple words is shaking of the Earth. It is a natural event, caused due to the release of energy, which
generates waves that travel in all directions. The release of energy occurs along a fault, a sharp break in the crustal rocks.

2nd Floor, 45 Pusa Road, Opp. Metro Pillar 128, Karol Bagh, New Delhi-110005
Ph: 08045248491, 07041021151 | Email: students@levelupias.com
• Statement 1 is not correct: Although body waves (P-waves and S-waves) are faster and arrive first at the surface
during an earthquake, they generally cause less damage compared
to surface waves. Body waves primarily cause damage by shaking
structures from within, while surface waves, which include Love
waves and Rayleigh waves, cause more significant shaking of the
Earth's surface and can lead to more structural damage.

• Statement 2 is correct: S-waves, also known as secondary wave or


shear waves, cannot travel through liquids. The fact that S-waves
disappear at the boundary between the outer and inner core
suggests that the outer core is in a liquid state. This discovery was
a major breakthrough in understanding Earth's internal structure.

• Statement 3 is correct: P-waves moves longitudinally i.e. the


propagation and vibration are in a same direction similar to the
sound waves. They are the fastest of all the earthquake waves.
They travel through gaseous, liquid and solid materials.
o S-waves propagate transversally i.e. the direction of
propagation and the direction of vibration is
perpendicular to each other.
Hence, option B is correct.

Q3. Consider the following statements with reference to Indian Ocean Dipole (IOD):
1. It is a phenomenon characterized by the fluctuation in sea surface temperature of the western and eastern tropical Indian
Ocean across equator.
2. A positive IOD results in abundant rainfall over Indonesia, southeast Asia and Australia.
3. An IOD phenomenon can influence an El Nino’s impact on the monsoon.
How many of the above statements is/are correct?
A. Only one
B. Only two
C. All three
D. None
Answer: B
Explanation:
• Statement 1 is correct: The Indian Ocean Dipole (IOD) is characterized by the temperature difference between the
western and eastern tropical Indian Ocean.
o When the sea surface temperature is greater than normal in western side (near the African coast) and less
than normal in the eastern side (near Indonesia), it is considered a Positive IOD.
o When the sea surface temperature is greater than normal in eastern side (near Indonesia) and less than
normal in the western side (near the African coast), it is considered a Negative IOD.
• Statement 2 is not correct: A positive IOD results in increased rainfall over the African coastline and the Indian sub-
continent, while suppressing rainfall over Indonesia, southeast Asia, and Australia. This is because the warmer
waters near Africa lead to rising air and enhanced convection, which in turn leads to increased rainfall over those
regions.

2nd Floor, 45 Pusa Road, Opp. Metro Pillar 128, Karol Bagh, New Delhi-110005
Ph: 08045248491, 07041021151 | Email: students@levelupias.com
• Statement 3 is correct: The IOD and El Nino are related phenomena, and they can interact to influence the Indian
monsoon. During El Nino, the Pacific side of Indonesia is cooler than normal, which can lead to a cooler Indian Ocean
side as well. This cooling can help in the development of a positive IOD, which in turn can offset some of the negative
impacts of El Nino on the Indian monsoon.
Hence, option B is correct.

Q4. Consider the following statements:


1. All the Latitudes on the Earth are of the same length and have equal distance between them, whereas the length of the
Longitude and distance between them decreases towards Pole.
2. Latitudes are used to classify heat zone of the Earth whereas Longitudes are used to measure Time.
Which of the statements given above is/are correct?
A. 1 only
B. 2 only
C. Both 1 and 2
D. Neither 1 nor 2
Answer: B
Explanation:
• Statement 1 is not correct: All parallel circles from the equator to the poles are called parallels of latitudes. They
are equidistant from each other. As we move away from the equator towards pole, the size of the parallels of latitudes
decreases (not same length).
o Longitudes on the other hand are imaginary semi-circles and the distance between them decreases steadily
poleward until it becomes zero at the poles, where all meridians meet.
• Statement 2 is correct: Latitudes are used to define different climate zones on Earth, such as the tropics, temperate
zones, and polar zones. Longitudes, on the other hand, are used to measure time, with each degree of longitude
corresponding to approximately 4 minutes of time.
Hence, option B is correct.

Q5. Consider the following statements with reference to the Salinity of the Ocean:
1. It is influenced by evaporation and ocean currents.
2. It can provide valuable information about precipitation patterns of a region.
3. The salinity of the Red Sea is higher than the Indian Ocean.
How many of the above statements is/are correct?
A. Only one
B. Only two
C. All three
D. None
Answer: C
Explanation:
• Statement 1 is correct: Ocean salinity is influenced by evaporation, precipitation, ocean currents, and other factors.
Evaporation increases salinity in areas where it's high, as more water evaporates leaving the salt behind. Conversely,
areas with high precipitation or freshwater influx from rivers will have lower salinity.
o Ocean currents can also affect salinity by transporting water with different salinity levels to other areas.
• Statement 2 is correct: Ocean salinity levels can fluctuate due to the volume of freshwater entering from precipitation
and land runoff. Areas with substantial precipitation and freshwater input typically exhibit lower salinity, whereas
arid regions with minimal precipitation have higher salinity.
2nd Floor, 45 Pusa Road, Opp. Metro Pillar 128, Karol Bagh, New Delhi-110005
Ph: 08045248491, 07041021151 | Email: students@levelupias.com
o Studying changes in ocean salinity allows us to infer alterations in precipitation patterns across different
regions and over time, providing valuable insights for climate research.
• Statement 3 is correct: The Red Sea is one of the most saline water bodies in the world, with a salinity ranging
between 36 and 38 parts per thousand (‰). This is higher than the average salinity of the Indian Ocean, which is
around 35 ‰. The high salinity of the Red Sea is due to its high evaporation rates and limited freshwater input.
Hence, option C is correct.

Q6. Consider the following statements with reference to the “Mass Movement”:
1. Weathering is a pre-requisite for mass movement.
2. It is aided by gravity and geomorphic agents like running water, glaciers, wind and waves have very negligible effect in the
process of mass movements.
Which of the statements given above is/are correct?
A. 1 only
B. 2 only
C. Both 1 and 2
D. Neither 1 nor 2
Answer: B
Explanation:
• Statement 1 is not corerct: Mass movements occur when the force of gravity causes rocks and debris to move
downhill, without the assistance of air, water, or ice. These movements can vary in speed and depth, including slow
processes like creep and rapid events like slides and falls. So, weathering is not a pre-requisite for mass movement,
though it aids mass movements.
• Statement 2 is corerct: Mass movements occur due to the force of gravity, without any involvement of agents like
water, ice, wind, or waves. This distinguishes mass movements from erosion, as there is no active erosion agent
involved in the process. Instead, materials shift due to gravity, with resistance from the materials themselves. Factors
such as weak unconsolidated materials, thinly bedded rocks, faults, steep slopes, abundant precipitation, torrential
rains, and sparse vegetation all contribute to the likelihood of mass movements occurring.
Hence, option B is correct.

Q7. Consider the following statements with reference to Lakes of India:


1. Kanwar Taal is Asia’s largest ox-bow lake, located in Bihar.
2. Badkhal Lake is a freshwater lake, located in Uttar Pradesh.
3. Wular Lake is a tectonic lake, located in Jammu and Kashmir.
4. Ashtamudi Lake is a brackish water lake, located in Tamil Nadu.
How many of the above statements is/are correct?
A. Only one
B. Only two
C. Only three
D. All four
Answer: B
Explanation:
• Statement 1 is correct: Kanwar Taal is the Asia's largest freshwater ox-bow lake, located in Begusarai district of Bihar.
It is also known as Kabar Taal. The lake's formation is attributed to the meandering of the Burhi Gandak River. It has
been designated as a Ramsar site.

2nd Floor, 45 Pusa Road, Opp. Metro Pillar 128, Karol Bagh, New Delhi-110005
Ph: 08045248491, 07041021151 | Email: students@levelupias.com
• Statement 2 is not correct: Badkhal Lake is a freshwater lake. It is located in Faridabad, Haryana (not in Uttar
Pradesh).
• Statement 3 is correct: Wular Lake is a tectonic lake, formed as a result of tectonic activity and the damming of the
Jhelum River, which flows through the Kashmir Valley. It is located in the state of Jammu and Kashmir.
• Statement 4 is not correct: Ashtamudi Lake is a brackish water lake, located in Kerala (not Tamil Nadu). It is second
largest after the Vembanad estuary system. It is known as Ashtamudi Kayal (meaning eight braids). It is a Ramsar
site.
Hence, option B is correct.

Q8. Consider the following statements with reference to land-use pattern in India:
1. Total Agricultural Land accounts for less than 30% of the total geographical area in India.
2. The net irrigated area in India is more than two-thirds of the total net sown area.
3. Per capita agriculture land in India is higher than the world average.
How many of the above statements is/are correct?
A. Only one
B. Only two
C. All three
D. None
Asnwer: D
Explanation:
• Statement 1 is not correct: According to the annual report (2022-2023) of Ministry of Agriculture & Farmers Welfare,
total agricultural land accounts for around 55% of the total geographical area in India.
o Also, as per the latest data from FAO and World Bank, total cultivable area accounts for more than 50% of
the total geographical area in India.
• Statement 2 is not correct: According to Economic Survey 2021-22, the share of net irrigated area accounts for about
49% of the total net sown area in the country and out of the net irrigated area, about 60% is irrigated through
groundwater and 40% is irrigated through canal systems.
• Statement 3 is not correct: Due to a huge population of the country, the per capita agriculture land (i.e. land suitable
for agriculture) is 0.12 hectares as against the world average of 0.29 hectares (as per Wastelands Atlas reports of
2019)
Hence, option D is correct.

Q9. “It is the main food crop in semi-arid areas of central and southern India. Maharashtra alone produces more than half of
the total production of the country. It is sown in both kharif and rabi seasons in southern states. But it is a kharif crop in
northern India”.
The above passage best describes which of the following crop?
A. Jowar
B. Bajra
C. Maize
D. Wheat
Answer: A
Explanation:
• Jowar is main food crop in semi-arid areas of central and southern India. Maharashtra alone produces more than
half of the total jowar production of the country. Other leading producer states of jowar are Karnataka, Madhya

2nd Floor, 45 Pusa Road, Opp. Metro Pillar 128, Karol Bagh, New Delhi-110005
Ph: 08045248491, 07041021151 | Email: students@levelupias.com
Pradesh and Andhra Pradesh. It is sown in both kharif and rabi seasons in southern states. But it is a kharif crop in
northern India where it is mostly grown as a fodder crop.
Hence, option A is correct.

Q10. Consider the following statements with reference to Subtropical westerly Jet Streams:
1. They are winds of upper air circulation that generally blow from west to east.
2. Their pattern can be altered by the presence of high mountains and highlands on the earth's surface.
3. These blow at a constant speed throughout the year.
4. They are limited to tropical latitudes with occasional movements in the sub-tropics.
How many of the above statements is/are correct?
A. Only one
B. Only two
C. Only three
D. All four
Answer: B
Explanation:
• Statement 1 is correct: Subtropical westerly Jet Streams are fast-moving bands of air found in the upper
troposphere, typically at altitudes of 9-12 km. They primarily blow from west to east. All of Western and Central Asia
remains under the influence of westerly winds. These winds blow across the Asian continent at latitudes north of the
Himalayas roughly parallel to the Tibetan highlands.
• Statement 2 is correct: The presence of high mountains and highlands can influence the flow and pattern of jet
streams. The terrain can cause the jet streams to meander or change direction.
o Tibetan highlands act as a barrier in the path of these jet streams. As a result, jet streams get bifurcated.
One of its branches blows to the north of the Tibetan highlands, while the southern branch blows in an
eastward direction, south of the Himalayas.
• Statement 3 is not correct: They are not always constant in speed. Their speed varies from about 110 km/h in
summer to about 184 km/h in winter.
• Statement 4 is not correct: They are not limited to tropical latitudes only. While they can dip into sub-tropics
occasionally, their main zone of influence lies between 30° and 60° latitude in both hemispheres.
Hence, option B is correct.

Q11. Consider the following statements with reference to the Madden Julian Oscillation (MJO):
1. It is a tropical intra-seasonal climate variability with a period ranging from 30-60 days.
2. It is a static phenomenon that causes changes in the pressure of the oceans across the world.
3. The convective phase of the MJO results in enhanced rainfall.
How many of the above statements is/are correct?
A. Only one
B. Only two
C. All three
D. None
Answer: B
Explanation:
• Statement 1 is correct: The Madden Julian Oscillation (MJO) is a tropical intra-seasonal climate phenomenon
characterized by its relatively short time scale, typically lasting 30 to 60 days. It involves variations in atmospheric
circulation, cloudiness, rainfall, and other parameters over the tropical regions of the Indian and Pacific Oceans.
2nd Floor, 45 Pusa Road, Opp. Metro Pillar 128, Karol Bagh, New Delhi-110005
Ph: 08045248491, 07041021151 | Email: students@levelupias.com
• Statement 2 is not correct: The MJO is not static but rather a traveling pattern that moves slowly eastwards over
the Indian and Pacific Oceans. It exhibits a planetary-scale organization and travels along the equator and can
influence weather patterns across large geographical areas.
• Statement 3 is correct: The MJO consists of two main phases: One is enhanced rainfall (or convective) phase and
the other is suppressed rainfall phase.
o During the convective phase, there is enhanced cloudiness and rainfall associated with deep convection,
leading to increased precipitation in the affected regions. This phase is characterized by active weather
patterns and atmospheric instability.
o These two phases produce opposite changes in clouds and rainfall, and this entire dipole (i.e., having two
main opposing centres of action) propagates eastwards.
Hence, option B is correct.

Q12. Consider the following statements with reference to El Nino:


1. It leads to increase in the temperature of water on the Peruvian coast.
2. It results in an increase in the amount of planktons in the Eastern Pacific ocean.
3. It can delay the onset of southwest monsoons in India.
4. It accentuates the global warming phenomenon and contributes to climate change.
How many of the above statements is/are correct?
A. Only one
B. Only two
C. Only three
D. All four
Answer: C
Explanation:
El Nino is complex weather patterns resulting from variations in ocean temperatures in the Equatorial Pacific Region. The
system involves oceanic and atmospheric phenomena with the appearance of warm currents off the coast of Peru in the
Eastern Pacific and affects weather in many places including India.
• Statement 1 is correct: El Nino is characterized by the warming of sea surface temperatures in the central and eastern
equatorial Pacific, including off the coast of Peru by 10°C.
• Statement 2 is not correct: During an El Nino situation, when the warm current appears, upwelling ceases and
phytoplankton production is greatly diminished and the fish move to colder regions farther south (or they starve).
• Statement 3 is correct: The changes in ocean temperatures and atmospheric circulation associated with El Nino can
weaken the monsoon winds, leading to a delayed or deficient monsoon season in India.
• Statement 4 is correct: While El Nino is a natural climate cycle, it can temporarily amplify global warming trends.
Additionally, the increased frequency and intensity of El Nino events in recent decades is partly attributed to human-
caused climate change.
Hence, option C is correct.

Q13. On which date does Delhi experience the sun directly overhead (the sun’s rays fall perpendicular) at noon?
A. 21 or 22 March
B. 21 or 22 June
C. 19 or 20 December
D. None
Answer: D
Explanation:
2nd Floor, 45 Pusa Road, Opp. Metro Pillar 128, Karol Bagh, New Delhi-110005
Ph: 08045248491, 07041021151 | Email: students@levelupias.com
• Sunshine is directly overhead in places which lie on or between the Tropic of Cancer and the Tropic of Capricorn.
All the regions which are not present between these two Tropics do not experience sunshine directly overhead at
noon.
• Delhi, located at 28°7' North latitude, lies north of the Tropic of Cancer. Therefore, Delhi will never experience the
sun directly overhead at noon throughout the year.
Hence, option D is correct.

Q14. Consider the following Ocean currents:


1. Peruvian
2. Oyashio
3. Canary
4. Benguela
5. Kuroshio
How many of the above currents are Cold currents?
A. Only two
B. Only three
C. Only four
D. All five
Answer: C
Explanation:
Ocean currents are continuous, directed movements of ocean water generated by various forces such as wind, temperature,
salinity, and the Earth's rotation. These currents play a crucial role in regulating global climate patterns, redistributing heat,
nutrients, and influencing weather systems.
• Cold Currents
are the currents
which brings
cold water into
warm water
areas and Warm
currents bring
warm water into
cold areas.
• Peruvian
(Humboldt),
Canary,
Benguela and
Oyashio are cold
currents
whereas
Kuroshio (Japan Current) are warm currents.

Hence, option C is correct.

Q15. Consider the following statements with reference to the course of a river:
1. In youth stage, V-shaped valleys are formed.
2nd Floor, 45 Pusa Road, Opp. Metro Pillar 128, Karol Bagh, New Delhi-110005
Ph: 08045248491, 07041021151 | Email: students@levelupias.com
2. In mature stage, lateral corrasion replaces vertical corrasion.
3. In old stage, there is complete absence of vertical and lateral corrasion.
4. In old stage, deposition is the main work of river.
How many of the above statements is/are NOT correct?
A. Only one
B. Only two
C. Only three
D. All four
Answer: A
Explanation:
• Statement 1 is correct: In the youth stage, the river has high velocity and erosive power due to steep gradient. This
leads to vertical corrasion, carving out a V-shaped valley.
• Statement 2 is correct: In the mature stage, as the gradient lessens, the river's velocity decreases. Vertical corrasion
slows down, and lateral corrasion (erosion of river banks) becomes more prominent. It widens the river channel,
leading to the formation of a wider valley.
• Statement 3 is not correct: In the old stage, vertical corrasion almost ceases in this stage though lateral corrasion
still goes on to erode its banks further.
• Statement 4 is correct: In the old stage, the river is often characterized by a gentle gradient and a broad, flat
floodplain. Deposition becomes a dominant process, leading to the accumulation of sediment and the formation of
features such as floodplains, deltas, meanders and oxbow lakes.
Hence, option A is correct.

Q16. Consider the following statements with reference to the Thermosphere:


1. In this layer, temperature rises very rapidly with increasing height.
2. Ionosphere is a part of this layer.
3. This layer helps in radio transmission.
4. All the communication satellites are launched in this layer only.
5. International Space Station (ISS), orbits the Earth within the thermosphere.
Which of the statements given above is/are correct?
A. 1, 3, 4 and 5 only
B. 2, 4 and 5 only
C. 1, 2, 3 and 5 only
D. 1, 2 and 3 only
Answer: C
Explanation:
Thermosphere:
The thermosphere is the layer of the Earth's atmosphere that lies just above the mesosphere and under the exosphere.
• Statement 1 is correct: In this layer, temperatures increase with altitude due to the absorption of high-energy solar
radiation by the few gas molecules present in this layer.
• Statement 2 is correct: The ionosphere, which contains charged particles (ions and electrons), is found within the
thermosphere. The ionosphere plays a crucial role in the reflection and absorption of radio waves, affecting radio
communication.
• Statement 3 is correct: The ionosphere within the thermosphere reflects radio waves, making it possible for long-
distance radio communication, especially during the day when the lower layers of the atmosphere do not support
long-range transmission.
2nd Floor, 45 Pusa Road, Opp. Metro Pillar 128, Karol Bagh, New Delhi-110005
Ph: 08045248491, 07041021151 | Email: students@levelupias.com
• Statement 4 is not correct: Communication satellites are generally launched into the geostationary orbit at a height
of 35,786 km, which is located in the exosphere, beyond the thermosphere.
• Statement 5 is correct: The International Space Station (ISS) is located within the thermosphere, between 330 and
435 km.
Hence, option C is correct.

Q17. Consider the following statements with reference to the world distribution of rainfall:
1. Rainfall goes on increasing from the Equator towards the Poles.
2. The coastal areas of the world receive greater amounts of rainfall than the interior of the continents.
3. The rainfall is more over the oceans than on the landmasses of the world.
How many of the above statements is/are correct?
A. Only one
B. Only two
C. All three
D. None
Answer: B
Explanation:
• Statement 1 is not correct: Equatorial regions generally experience high and consistent precipitation due to the
convergence of trade winds and the presence of the Inter-Tropical Convergence Zone. The intensity of rainfall
decreases as one moves away from the equator towards pole.
• Statement 2 is correct: Coastal areas often receive more rainfall because of the proximity to oceans. Oceans act as
a moisture source, and when air masses move from the ocean to the land, they may release their moisture in the
form of rainfall. This phenomenon contributes to higher rainfall in coastal regions compared to the interior of
continents, which may be farther away from moisture sources.
• Statement 3 is correct: The rainfall is more over the oceans than on the landmasses of the world because of being
great sources of water. It contributes significantly to global rainfall as they serve as vast reservoirs of water. Solar
radiation heats the ocean surface, causing water to evaporate and rise into the atmosphere as moisture-laden air.
Hence, option B is correct.

Q18. "These are the ports which originally developed as calling points on main sea routes where ships used to anchor for
refuelling, watering and taking food items. Later on, they developed into commercial ports."
The above passage best describes which of the following ports?
A. Out Ports
B. Entrepot Ports
C. Ferry Ports
D. Ports of Call
Answer: D
Explanation:
Ports of Call:
• These are the ports which originally developed as calling points on main sea routes where ships used to anchor for
refuelling, watering and taking food items. Later on, they developed into commercial ports.
• For example: Aden, Honolulu and Singapore.
Hence, option D is correct.

Addition Info:
2nd Floor, 45 Pusa Road, Opp. Metro Pillar 128, Karol Bagh, New Delhi-110005
Ph: 08045248491, 07041021151 | Email: students@levelupias.com
• Out Ports: These are deep water ports built away from the actual ports. These serve the parent ports by receiving
those ships which are unable to approach them due to their large size. For example, Athens and its out-port Piraeus
in Greece.
• Entrepot Ports: These are collection centers where the goods are brought from different countries for export.
Singapore is an Entrepot for Asia. Rotterdam for Europe and Copenhagen for the Baltic region.
• Packet Station or Ferry ports: These are exclusively concerned with the transportation of passengers and mail across
water bodies covering short distances. These stations occur in pairs located in such a way that they face each other
across the water body, e.g. Dover in England and Calais in France across the English Channel.

Q19. Consider the following pairs:


Cities/Towns Located on the bank of River
1. Lucknow Gomti
2. Ujjain Shipra
3. Nashik Krishna
4. Ganjam Rushikulya
5. Bharuch Narmada
How many of the above pairs are correct?
A. Only two
B. Only three
C. Only four
D. All five
Answer: C
Explanation:
• Pair 1 is correct: Lucknow is situated on the banks of the Gomti River in the state of Uttar Pradesh. Jaunpur is also
located on this river. It is a left bank tributary of Ganga River.
• Pair 2 is correct: Ujjain is located on the banks of the Shipra River in the state of Madhya Pradesh. The Shipra River
is a tributary of the Chambal River.
• Pair 3 is not correct: Nashik is situated on the banks of the Godavari River (not Krishna River) in the state of
Maharashtra.
• Pair 4 is correct: Ganjam is located on the banks of the Rushikulya River in the state of Odisha. It originates in the
Eastern Ghats and drains into the Bay of Bengal. It is also an important habitat for diverse aquatic life, including Olive
Ridley turtles, which nest along the river's mouth.
• Pair 5 is correct: Bharuch is situated on the banks of the Narmada River, in the state of Gujarat. It is a major industrial
city and is known for its chemical and textile industries. Jabalpur is also located on the bank of Narmada River.
Hence, option C is correct.

Q20. Arrange the following passes from North to South:


1. Zojila Pass
2. Rohtang Pass
3. Bara Lacha Pass
4. Mintaka
Select the correct answer using the code given below:
A. 4-1-3-2
B. 1-4-3-2
C. 4-1-2-3
2nd Floor, 45 Pusa Road, Opp. Metro Pillar 128, Karol Bagh, New Delhi-110005
Ph: 08045248491, 07041021151 | Email: students@levelupias.com
D. 1-4-2-3
Answer: A
Explanation:
The correct order (North to South):
• Mintaka Pass
• Zojila Pass
• Bara Lacha Pass
• Rohtang Pass
Hence, option A is correct.

Q21. Consider the following statements:


Statement-I:
An observer standing at sea level near the poles are closer to the center of Earth than people standing at sea level near the
Equator.
Statement-II:
A rotating body tends to form an oblate spheroid rather than a sphere.
Which one of the following is correct in respect of the above statements?
A. Both Statement-I and Statement-II are correct and Statement-II is the correct explanation for Statement-I.
B. Both Statement-I and Statement-II are correct and Statement-II is not the correct explanation for Statement-I.
C. Statement-I is correct but Statement-II is incorrect.
D. Statement-I is incorrect but Statement-II is correct.
Answer: A
Explanation:
• Statement-I is correct: The Earth is not a perfect sphere, but slightly oblate due to its rotation. This means the Earth
bulges slightly at the equator and is flattened at the poles. As a result, people standing at sea level near the poles
are closer to the center of Earth than those standing at the equator.
• Statement-II is correct: When a rotating body, like the Earth, spins, centrifugal force acts outwards at the equator,
trying to stretch it, while gravity pulls inwards from all directions. This tug-of-war between forces results in the body
bulging at the equator and flattening at the poles, forming an oblate spheroid.
2nd Floor, 45 Pusa Road, Opp. Metro Pillar 128, Karol Bagh, New Delhi-110005
Ph: 08045248491, 07041021151 | Email: students@levelupias.com
Hence, option A is correct.

Q22. ‘Ponmudi hills’, recently seen in news, is located in which of the following states?
A. Andhra Pradesh
B. Karnataka
C. Telangana
D. Kerala
Answer: D
Explanation:
• Ponmudi hills are also known as ‘Golden Hill’ or ‘Golden Peak’. It is located in the state of Kerala. It is part of the
Western Ghats Mountain range. The hills are a part of the Agasthayamalai landscape, at the southernmost tip of the
Western Ghats. These hills are rich in biodiversity.
• Recently, the species of damselfly has been discovered from this region. It is named Cliffside Bambootail. It lays eggs
in the moss beds over rock cliffs.
Hence, option D is correct.

Q23. Consider the following pairs:


Type of precipitation Description
1. Sleet Precipitation below 0 degree Celsius
2. Hail Frozen raindrops and refrozen melted snow-water
3. Snow Small rounded solid pieces of ice
How many of the pairs given above is/are correctly matched?
A. Only one
B. Only two
C. All three
D. None
Answer: D
Explanation:
• Pair 1 is not correct: Sleet - When a layer of warm air sits above a sub-freezing layer near the ground, precipitation
falls as sleet. Raindrops, leaving the warmer air, freeze as they encounter the colder air below, forming small pellets
of ice that reach the ground. These pellets are no larger than the original raindrops.
• Pair 2 is not correct: Hail - Hailstones are formed when raindrops freeze into small, round pieces of ice as they pass
through colder layers of the atmosphere. These icy pellets have multiple concentric layers, giving them their
characteristic structure.
• Pair 3 is not correct: Snow - When the temperature drops below 0 degrees Celsius, moisture in the air freezes and
falls to the ground as snowflakes, a phenomenon known as snowfall. These hexagonal crystals join together to form
snowflakes.
Hence, option D is correct.

Q24. Consider the following statements with reference to “Savanna landscape”:


1. It is typified by long trees and short grass.
2. The trees are deciduous.
3. Trees have broad trunks, with water storing devices to survive through the prolonged drought.
How many of the above statements is/are correct?
A. Only one
2nd Floor, 45 Pusa Road, Opp. Metro Pillar 128, Karol Bagh, New Delhi-110005
Ph: 08045248491, 07041021151 | Email: students@levelupias.com
B. Only two
C. All three
D. None
Answer: B
Explanation:
• Statement 1 is not correct: The savanna landscape is characterized by its tall grasses and scattered, short trees. This
is different from tropical grasslands, where trees are generally absent or sparse. Example - Elephant grass.
• Statement 2 is correct: Many trees in the savanna are deciduous, meaning they shed their leaves during the dry
season. This adaptation helps them conserve water during periods of drought, reducing water loss through
transpiration.
• Statement 3 is correct: Some trees in the savanna, like baobabs and bottle trees, have adaptations such as broad
trunks or specialized structures for storing water. These adaptations help them survive through prolonged periods of
drought when water is scarce.
Hence, option B is correct.

Q25. Consider the following statements with reference to 'Block mountains:


1. They are formed due to faulting in the earth crust.
2. They are the most widespread mountains in the world.
3. Aravalli mountain range is one of the oldest block mountains of the world.
How many of the above statements is/are correct?
A. Only one
B. Only two
C. All three
D. None
Answer: A
Explanation:
• Statement 1 is correct: Block mountains are formed due to faulting in the Earth's crust. Faulting occurs when there
is movement along a fracture (fault) in the Earth's crust, causing blocks of rock to move vertically. As these blocks
move, they can be uplifted, creating mountainous features known as block mountains.
• Statement 2 is not correct: Fold mountains (not Block mountains), which are formed through the folding of layers of
rock due to tectonic compression, are more widespread in the world.
o Examples of fold mountains include the Himalayas, the Andes, and the Alps.
• Statement 3 is not correct: The Aravalli Range in Northwestern India is one of the oldest fold mountains (not block
mountains) ranges in the world. Aravalli Range has undergone significant folding and uplift over millions of years due
to tectonic activity, making it a prominent example of fold mountains, not block mountains.
Hence, option A is correct.

Q26. Consider the following statements:


1. The age of oceanic crust increases with distance from mid-ocean ridges.
2. An oceanic crust is formed at mid-ocean ridges and is destroyed in the subduction zone.
3. The sediments on the ocean floor are thinner than the sediments on the continents.
How many of the above statements is/are NOT correct?
A. Only one
B. Only two
C. All three
2nd Floor, 45 Pusa Road, Opp. Metro Pillar 128, Karol Bagh, New Delhi-110005
Ph: 08045248491, 07041021151 | Email: students@levelupias.com
D. None
Answer: D
Explanation:
• Statement 1 is correct: The mid-oceanic ridges are areas where new oceanic crust is formed through volcanic activity.
As molten rock rises and solidifies at these ridges, it creates new oceanic crust. Therefore, the rocks closest to the
mid-oceanic ridges are the youngest and age of oceanic crust increases with distance from mid-ocean ridges.
• Statement 2 is correct: Seafloor spreading at mid-ocean ridges creates new oceanic crust. However, this crust is
constantly being recycled back into the mantle at subduction zones, where it dives under another tectonic plate.
• Statement 3 is correct: Due to the constant movement and subduction of oceanic plates, sediments on the ocean
floor don't have the same opportunity to accumulate over time as they do on the more stable continental masses.
Therefore, continental sediments tend to be much thicker than their oceanic counterparts.
Hence, option D is correct.

Q27. Consider the following statements with reference to Intensive subsistence agriculture:
1. In this type of agriculture, the yield per unit area is high, but per capita labour productivity is low.
2. It is usually practiced where the land holdings are very small due to the high density of population.
3. This type of agriculture is characterized by the dominance of the Cash crops.
How many of the above statements is/are NOT correct?
A. Only one
B. Only two
C. All three
D. None
Answer: A
Explanation:
• Statement 1 is correct: In intensive subsistence agriculture, the yield per unit area is high, farmers focus on
maximizing productivity on small land holdings through various intensive farming practices. However, the per capita
labor productivity is low as the focus is on producing enough food for the family rather than for the market.
• Statement 2 is correct: In areas where intensive subsistence agriculture is practiced, the population density is
typically high. As a result, land is divided into small plots among numerous families, and each family cultivates a
small piece of land intensively to meet their subsistence needs.
• Statement 3 is not correct: Intensive subsistence agriculture primarily involves the cultivation of food crops (not
Cash crops) to meet the needs of the farmer's family. The focus is on growing crops for personal consumption rather
than for sale in the market.
Hence, option A is correct.

Q28. The drainage pattern of an area, depends on which of the following factors?
1. Time
2. Structure of rocks
3. Topography
4. Amount and periodicity of water flow
Select the correct answer using the code given below:
A. 1, 2 and 3 only
B. 1 and 2 only
C. 3 and 4 only
D. 1, 2, 3 and 4
2nd Floor, 45 Pusa Road, Opp. Metro Pillar 128, Karol Bagh, New Delhi-110005
Ph: 08045248491, 07041021151 | Email: students@levelupias.com
Answer: D
Explanation:
• Statement 1 is correct: Time - The length of time over which a river or stream has been flowing in its current path can
affect the drainage pattern. Over time, the flow of water can erode the land and create specific patterns.
• Statement 2 is correct: Structure of rocks - The type and structure of rocks in an area can influence the drainage
pattern. For example, rocks that are easily eroded can lead to the formation of valleys and channels that affect how
water flows.
• Statement 3 is correct: Topography - The shape and features of the land, such as mountains, valleys, and plains,
determine how water flows across the landscape. This can influence the direction and pattern of drainage.
• Statement 4 is correct: Amount and periodicity of water flow - The volume of water flowing through an area, as well
as the frequency of this flow (such as seasonal variations), can impact the drainage pattern. Higher water flow can
lead to more erosion and the formation of different drainage patterns.
Hence, option D is correct.

Q29. Consider the following statements with reference to the Extra Tropical Cyclones:
1. They form along the Polar front.
2. They can originate over sea only.
3. They affect much smaller area, as compared to the tropical cyclone.
How many of the above statements is/are correct?
A. Only one
B. Only two
C. All three
D. None
Answer: A
Explanation:
• Statement 1 is correct: A polar front is a weather front boundary between the polar cell and the Ferrel cell located
at 60-degree latitude in both hemispheres.
o Extra-tropical cyclones form along the polar front, which is a boundary separating cold polar air from
warmer air from lower latitudes. This contrast in air masses leads to the formation of a frontal zone where
weather disturbances can develop into cyclones.
• Statement 2 is not correct: Extra-tropical cyclones can originate over both land and sea. While they often develop
over the ocean where there is more moisture and energy available, they can also form over land, especially when
there is a strong temperature contrast or other favorable conditions present.
• Statement 3 is not correct: Extra-tropical cyclones typically affect a larger area than tropical cyclones. This is because
they are often associated with fronts that extend over hundreds of kilometers, leading to widespread precipitation
and wind patterns.
o Tropical cyclones, on the other hand, are more compact and tend to have a more localized impact, though
they can still cause significant damage over a smaller area.
Hence, option A is correct.

Q30. Consider the following statements:


1. In the wild state, it is a perennial plant however most of the cultivated crops are annual.
2. It has a tap root system with secondary roots that branches laterally from the primary root.
3. Warm days and cool nights during the period of fruiting are conducive for development.
Which of the following crop is being described in the above statements?
2nd Floor, 45 Pusa Road, Opp. Metro Pillar 128, Karol Bagh, New Delhi-110005
Ph: 08045248491, 07041021151 | Email: students@levelupias.com
A. Sugarcane
B. Cotton
C. Rice
D. Millet
Answer: B
Explanation:
Cotton:
• In its wild state, cotton is a perennial plant, meaning it can live for several years and can reach a height of 5-6m.
However, in agriculture, cotton is typically grown as an annual crop, meaning it completes its life cycle (from
germination to seed production) in one growing season.
• Cultivated cotton is a herbaceous plant that can attain a height of 75-200 cm. The cotton plant has a taproot system
with lateral secondary roots branching from the primary root. The main stem is erect and heavily branched, with
branches developing from buds at the stem nodes.
• In India, cotton is extensively cultivated in the states of Gujarat, Maharashtra, Karnataka, Madhya Pradesh, Punjab,
Rajasthan, Haryana, Tamil Nadu, and Uttar Pradesh.
• Cotton thrives in tropical and subtropical climates. It requires a minimum temperature of 15°C for optimal
germination in the field. The best temperature range for vegetative growth is 21-27°C, and the plant can tolerate
temperatures up to 43°C. However, temperatures below 21°C are harmful to the crop. Cotton plants benefit from
warm days and cool nights with significant temperature fluctuations during the fruiting period, as this promotes the
development of cotton bolls and fibers.
• Cotton can be grown in a variety of soils, ranging from well-drained deep alluvial soils in the north to black clayey
soils of varying depths in the central region, and in black and mixed black and red soils in the south. Cotton plants are
moderately tolerant to salinity but sensitive to waterlogging, so they prefer well-drained soils.
Hence, option B is correct.

Q31. Consider the following statements with reference to Peaty soils of India:
1. They are generally found in areas that have high humidity and precipitation.
2. They are low in humus and organic content.
3. They are heavy, black and have high acidic content.
How many of the above statements is/are correct?
A. Only one
B. Only two
C. All three
D. None
Answer: B
Explanation:
• Statement 1 is correct: Peaty soils are often found in areas with high humidity and precipitation because these
conditions promote the accumulation of organic material. Peat is formed from the partial decomposition of plant
material in waterlogged conditions.
o Found in Kerala backwaters, Sunderbans of West Bengal, and coastal areas of Odisha and Tamil Nadu.
• Statement 2 is not correct: Peaty soils are rich in humus and organic matter. These types of soils constitute about 10
to 40% of the organic matter and also a reasonable amount of soluble salts.
• Statement 3 is correct: Peaty soils are heavy in texture due to the high organic content. The black color is a result of
the presence of organic matter. Additionally, peaty soils tend to be acidic because the decomposition of organic
material produces acidic byproducts.
2nd Floor, 45 Pusa Road, Opp. Metro Pillar 128, Karol Bagh, New Delhi-110005
Ph: 08045248491, 07041021151 | Email: students@levelupias.com
Hence, option B is correct.

Q32. Consider the following statements with reference to Thunderstorms:


1. They are caused by intense convection on moist hot days.
2. They are a well-grown cirrocumulus cloud.
3. They are characterised by intense updraft of rising warm air.
How many of the above statements is/are correct?
A. Only one
B. Only two
C. All three
D. None
Answer: B
Explanation:
• Statement 1 is correct: Thunderstorms are caused by intense convection, which is the rapid upward movement of
air parcels. Warm, moist air near the Earth's surface rises due to buoyancy, cools as it ascends, condenses, and forms
clouds. As the water vapor condenses, it releases heat, which further fuels the updraft, creating a self-sustaining cycle.
• Statement 2 is not correct: Cirrocumulus clouds are small, white puff clouds that are composed of ice crystals and
are found at high altitudes.
o Thunderstorms, on the other hand, are associated with tall, cumulonimbus clouds that can reach up to
several miles in height and are composed of water droplets at lower altitudes and ice crystals at higher
altitudes.
• Statement 3 is correct: Intense updrafts of warm, moist air are a key characteristic of thunderstorms. This updraft
is what fuels the storm's development and leads to the various weather phenomena associated with thunderstorms,
such as heavy rain, hail, lightning, and strong winds.
Hence, option B is correct.

Q33. Consider the following statements with reference to the ‘Theory of plate tectonics’:
1. A tectonic plate is a massive, irregularly-shaped slab of solid rock, generally composed of both continental and oceanic
lithosphere.
2. In divergent plate boundaries, the oceanic crust is destroyed.
3. In convergent plate boundaries, new crust is generated.
4. In transform plate boundaries, the crust is neither produced nor destroyed.
5. Movement of the plates is caused by pole fleeing force and tidal force.
How many of the above statements are correct?
A. Only two
B. Only three
C. Only four
D. All five
Answer: A
Explanation:
• Statement 1 is correct: A tectonic plate (also called lithospheric plate) is a massive, irregularly-shaped slab of solid
rock, generally composed of both continental and oceanic lithosphere.
• Statement 2 is not correct: In divergent plate boundaries, new crust is generated as the plates pull apart and magma
rises from the mantle to fill the gap.

2nd Floor, 45 Pusa Road, Opp. Metro Pillar 128, Karol Bagh, New Delhi-110005
Ph: 08045248491, 07041021151 | Email: students@levelupias.com
• Statement 3 is not correct: In convergent plate boundaries, the crust is destroyed as the two plates move toward
each other, and one plate is forced beneath the other in a process called subduction.
• Statement 4 is correct: In transform plate boundaries, tectonic plates slide past each other horizontally, and while
earthquakes can occur, there is no significant creation or destruction of crust.
• Statement 5 is not correct: The movement of tectonic plates is primarily driven by mantle convection currents
beneath the Earth's surface. Heat from the interior causes material in the mantle to rise, and as it cools, it sinks back
down. This process sets up convection currents that drive the movement of tectonic plates.
o Heat within the earth comes from two main sources: radioactive decay and residual heat.
Hence, option A is correct.

Q34. “They are the oldest inhabitants of southern Africa. Their home is in the vast expanse of the Kalahari desert. They are
the remnants of Africa's oldest cultural group, genetically the closest surviving people to the original Homo sapiens ‘core’ from
which the Negroid people of Africa emerged. They are small in stature generally with light yellowish skin, which wrinkles very
early in life.”
The above passage best describes which of the following tribes?
A. Tuareg
B. Bushmen
C. Bedouin
D. Masai
Answer: B
Explanation:
Bushmen Tribes:
• The Bushmen, or San people, are indigenous hunter-gatherer people of southern Africa, particularly found in the
Kalahari Desert region. They are believed to have inhabited this area for thousands of years, making them one of the
oldest known populations in Africa.
• They are genetically considered to be among the closest living relatives to the original Homo sapiens, the ancestors
of all modern humans. This genetic closeness is one reason they are often referred to as the "oldest cultural group"
in Africa.
• Physically, they are often characterized by their relatively small stature and light yellowish skin tone, which can
appear wrinkled at an early age. These physical features are believed to be adaptations to their desert environment.
• They traditionally lived as hunter-gatherers, relying on hunting wild game and gathering wild plants for their
subsistence. Their traditional way of life has faced numerous challenges in recent centuries, including encroachment
on their land, loss of traditional hunting grounds, and cultural assimilation pressures from surrounding populations.
Hence, option B is correct.

Q35. Consider the following pairs:


Glaciers Location
1. Milam Himachal Pradesh
2. Zemu Sikkim
3. Pindari Uttarakhand
4. Shafat Ladakh

How many of the above pairs is/are correctly matched?


A. Only one
B. Only two
2nd Floor, 45 Pusa Road, Opp. Metro Pillar 128, Karol Bagh, New Delhi-110005
Ph: 08045248491, 07041021151 | Email: students@levelupias.com
C. Only three
D. All four
Answer: C
Explanation:
• Pair 1 is not correct: Milam glacier is situated 60km from Munsiyari, Uttarakhand. It is the source of the Goriganga
River.
• Pair 2 is correct: Zemu Glacier is located in Eastern Himalayas in Sikkim.
• Pair 3 is correct: Pindari glacier is located in the Trishul-Nanda Devi area of Kumaun Himalayas in the state of
Uttarakhand.
• Pair 4 is correct: The Shafat Glacier- Parkachik Glacier is a 14 kilometers long glacier in the Himalayan range in Ladakh,
India. The melting water flows into the Suru River, a tributary of the Indus.
Hence, option C is correct.

Q36. “Stalactites, Stalagmites and Uvalas” are characteristic landforms of which of the following types of topography?
A. Glacial topography
B. Fluvial topography
C. Desert topography
D. Karst topography
Answer: D
Explanation:
Karst topography:
• Karst topography is a landscape formed by the dissolution of soluble rocks such as limestone, dolomite, and gypsum.
The dissolution occurs primarily through the chemical reaction between the rock and slightly acidic rainwater or
groundwater.
o Stalactites are icicle-shaped formations that hang from the ceilings of caves, caverns, or other underground
spaces. They form as mineral-laden water drips from the ceiling, leaving behind mineral deposits as the
water evaporates.
o Stalagmites, on the other hand, are formations that grow up from the floor of caves, resulting from the
accumulation of mineral deposits left behind by dripping water. Stalagmites and stalactites often form in
tandem.
o Uvalas are large, enclosed depressions or sinkholes in the landscape, which are characteristic of karst
regions. These depressions are often created as the soluble rock dissolves over time, leading to the collapse
of the surface layer.
Hence, option D is correct.

Q37. Consider the following:


1. Nokrek Biosphere reserve
2. Achanakamar Amarkantak Biosphere reserve
3. Similipal Biosphere reserve
4. Panchmarhi Biosphere reserve
How many of the above biosphere reserves is/are located south of Tropic of Cancer?
A. Only one
B. Only two
C. Only three
D. All four
2nd Floor, 45 Pusa Road, Opp. Metro Pillar 128, Karol Bagh, New Delhi-110005
Ph: 08045248491, 07041021151 | Email: students@levelupias.com
Answer: C
Explanation:
• Achanakamar-Amarkantak, Similipal and Panchmarhi Biosphere reserves are located south of Tropic of Cancer.
• Nokrek is located north of Tropic of Cancer.
Hence, option C is correct.

Q38. Consider the following statements:


Statement-I:
In some parts of Southern India, Rice can be grown during any period of the year.
Statement-II:
Southern India receives continuous but light rainfall from western disturbances throughout the year.
Which one of the following is correct in respect of the above statements?
A. Both Statement-I and Statement-II are correct and Statement-II is the correct explanation for Statement-I.
B. Both Statement-I and Statement-II are correct and Statement-II is not the correct explanation for Statement-I.
C. Statement-I is correct but Statement-II is incorrect.
D. Statement-I is incorrect but Statement-II is correct.
Answer: C
Explanation:
• Statement-I is correct: In some parts of Southern India, rice can be grown during any period of the year due to the
region's high temperatures and the presence of irrigation facilities. Unlike in Northern India, where distinct Kharif,
Rabi, and Zaid seasons limit when certain crops can be grown, Southern India's tropical climate allows for year-round
cultivation of crops like rice. Additionally, the favorable agro-climatic conditions in Southern India support the
cultivation of some Kharif crops, like rice, thrice in a year.
• Statement-II is not correct: Western disturbances appear in the winter season in India and withdraw when Monsoon
comes. They are not responsible for majority of rainfall in South India. They become more important in the Northern
regions.
Hence, option C is correct.

2nd Floor, 45 Pusa Road, Opp. Metro Pillar 128, Karol Bagh, New Delhi-110005
Ph: 08045248491, 07041021151 | Email: students@levelupias.com
Q39. Consider the following statements with reference to Rip Current:
1. It is a strong flow of water running from a beach back to the open ocean, sea, or lake.
2. It can occur in areas with hard-bottom (rocky) or soft-bottom (sand or silt) beach topography.
3. It can form in a gap between sandbars, piers, or parts of a reef.
How many of the above statements is/are correct?
A. Only one
B. Only two
C. All three
D. None
Answer: C
Explanation:
• Statement 1 is correct: Rip currents are one of the most dangerous natural hazards in the world. A rip current is a
strong flow of water running from a beach back to the open ocean, sea, or lake. They can be more than 45 meters
(150 feet) wide, but most are less than 9 meters (30 feet). They can move at 8 kilometers (5 miles) per hour.
• Statement 2 is correct: Rip currents are formed by a beach topography. It can occur anywhere waves break on a
shore, regardless of whether the bottom is sandy, rocky, or a mix. The key factor is the way waves interact with the
underwater topography, creating channels for the rip current to flow through.
• Statement 3 is correct: Rip currents can form in different ways, including in gaps between sandbars, piers, or parts
of a reef. These features can influence the direction and strength of the rip current.
o For example, if waves are breaking over a sandbar, the water can flow back to the ocean through a gap in the
sandbar, creating a rip current.
Hence, option C is correct.

Q40. Consider the following statements with reference to the Western Cyclonic Disturbances:
1. These are tropical cyclones, which generate over the Mediterranean Sea.
2. The western disturbances are steered in India by the Westerly Jet Streams.
Which of the statements given above is/are correct?
A. 1 only
B. 2 only
C. Both 1 and 2
D. Neither 1 nor 2
Answer: B
Explanation:
• Statement 1 is not correct: The Western Cyclonic Disturbances are temperate cyclones (not tropical cyclones). They
originate in the Mediterranean region and move eastwards towards India, influencing the weather patterns in the
region.
• Statement 2 is correct: Western disturbances are guided and influenced by the Westerly Jet Streams, which are high-
altitude, fast-flowing air currents moving from west to east. The interaction between these disturbances and the jet
streams plays a crucial role in shaping weather conditions, particularly in the northern parts of India. An increase in
the prevailing night temperature generally indicates an advance in the arrival of these cyclonic disturbances.
Hence, option B is correct.

Q41. Consider the following statements with reference to the Desert region of India:
1. Chemical weathering is the most active degradation process in this region.
2. It is believed that during the Mesozoic era, this region was under the sea.
2nd Floor, 45 Pusa Road, Opp. Metro Pillar 128, Karol Bagh, New Delhi-110005
Ph: 08045248491, 07041021151 | Email: students@levelupias.com
3. The underlying rock structure of the desert is an extension of the Peninsular plateau.
How many of the above statements is/are correct?
A. Only one
B. Only two
C. All three
D. None
Answer: B
Explanation:
• Statement 1 is not correct: Physical weathering and wind action (not Chemical weathering) are the most active
degradation process in this region. For chemical weathering to be active, there is a need of hot and humid conditions.
Since the average rainfall is less than 15 cm, the dominant form of weathering is physical.
• Statement 2 is correct: It is believed that during the Mesozoic era, this region was under the sea. This can be
corroborated by the evidence available at wood fossils park at Aaka land marine deposits around Brahmsar, near
Jaisalmer (The approximate age of the wood-fossils is estimated to be 180 million years.
• Statement 3 is correct: The underlying rock structure of the desert is an extension of the Peninsular plateau.
Hence, option B is correct.

Q42. Consider the following rivers:


1. Manjira
2. Tungbhadra
3. Bhima
4. Koyna
How many of the above rivers are the tributaries of River Krishna?
A. Only one
B. Only two
C. Only three
D. All four
Answer: C
Explanation:
The Krishna is the second-largest east-flowing river of the Peninsula. It rises at Mahabaleshwar at an altitude of 1336 m
near the Jor village in the extreme north of district Satara, Maharashtra in the west, and meets the Bay of Bengal in Andhra
Pradesh, on the east coast.
• Right bank tributaries: Venna, Koyna, Panchganga, Dudhganga, Ghataprabha, Malaprabha and Tungabhadra are the
major right-bank tributaries
• Left bank tributaries: Bhima, Dindi, Peddavagu, Halia, Musi, Paleru, and Munneru are the major left-bank tributaries
The Manjira River is a tributary of the river Godavari. It passes through the states of Maharashtra, Karnataka and Telangana.
It originates in the Balaghat range of hills near the Ahmednagar district at an altitude of 823m and empties into the Godavari
River.
Hence, option C is correct.

Q43. Consider the following rivers:


1. Satluj
2. Kosi
3. Subansiri
How many of the above rivers is/are antecedent?
2nd Floor, 45 Pusa Road, Opp. Metro Pillar 128, Karol Bagh, New Delhi-110005
Ph: 08045248491, 07041021151 | Email: students@levelupias.com
A. Only one
B. Only two
C. All three
D. None
Answer: C
Explanation:
An antecedent river is one that maintains its course and continues to flow in the same general direction as it did before the
land was uplifted or the surrounding area was otherwise changed by geological processes. In other words, these rivers existed
before the land features around them changed, and they maintained their original path despite these changes.
• Satluj: The Satluj River is an antecedent river because it flows almost parallel to the Indus for about 400 km before
entering India. It maintained its course even as the Himalayas rose around it, cutting through the mountain range
to create a gorge at Rupar. The river is significant because it feeds the canal system of the Bhakra Nangal project.
• Kosi: The Kosi River is also an antecedent river. Its source is to the north of Mount Everest in Tibet, where its
mainstream Arun rises. Despite crossing the Central Himalayas in Nepal and being joined by the Son Kosi and Tamur
Kosi rivers, it has maintained its course. It forms Sapt Kosi after uniting with the river Arun.
• Subansiri: The Subansiri River, a tributary of the Brahmaputra, originates in Tibet. Like the Satluj and Kosi, it is
considered an antecedent river because it has maintained its course despite the geological changes in the region.
Hence, option C is correct.

Q44. Consider the following statements with reference to the Peninsular drainage system:
1. It is younger than the Himalayan drainage system.
2. All Peninsular rivers flow from west to east direction.
3. The Peninsular rivers frequently change their course because of heavy meandering.
How many of the above statements is/are correct?
A. Only one
B. Only two
C. All three
D. None
Answer: D
Explanation:
• Statement 1 is not correct: The Peninsular drainage system is older than the Himalayan drainage system. Evidence
such as broader, shallower valleys and mature river characteristics point to its greater age.
• Statement 2 is not correct: While most Peninsular rivers flow west to east, some exceptions exist. Narmada and Tapi
flow from east to west.
• Statement 3 is not correct: The Peninsular Rivers are characterised by fixed course, absence of meanders and non-
perennial flow of water. They flow through hard rock terrain, leading to a more fixed course with less meandering.
Hence, option D is correct.

Q45. Consider the following factors:


1. Fast moving cold air.
2. Stubble burning in states of Haryana and Punjab.
3. Geographical position of Delhi surrounded by the landmass.
How many of the above act as aggravating factors for Winter pollution in Delhi?
A. Only one
B. Only two
2nd Floor, 45 Pusa Road, Opp. Metro Pillar 128, Karol Bagh, New Delhi-110005
Ph: 08045248491, 07041021151 | Email: students@levelupias.com
C. All three
D. None
Answer: B
Explanation:
While Delhi has been in the one of most polluted cities globally according to World Health Organisation, the Winter pollution
in Delhi is even more severe. As the Winter descends on Delhi, there is more than twice air pollution (especially PM2.5) than
during the spring and summer.
• Statement 1 is not correct: During the winter months cool air stagnates over the city, keeping pollution close to the
ground where people breathe. Delhi’s persistent winter fog worsens this problem. Stagnant air also explains why
pollution levels vary less widely over the day in winter than in spring.
• Statement 2 is correct: One of the major factors for the same include stubble burning in the states of surrounding
Punjab, Uttar Pradesh, Haryana and Rajasthan from late September to early November. This is the cheapest method
for them to prepare their fields for the rabi or winter crop. Consequently, this stubble burning generates pollution
across these states, eventually permeating through the Indo-Gangetic plain.
• Statement 3 is correct: Also, Delhi is cursed with poor geography. It is located at the head of the Indo-Gangetic plain,
north east of the Thar Desert, to the north-west of the central plains and to the south-west of the Himalayas. As
winds arrive from the coasts, bringing with them pollutants picked up along the way, they get ‘trapped’ right before
the Himalayas. The air pressure pushes from one direction, and with the inability to escape quickly in the other, the
particulate matter accumulates over the northern plains. Thus, the Himalayas prevent polluted air from escaping to
the north creating the so called “valley effect”.
Hence, option B is correct.

Q46. Arrange the following ranges in the North to South direction:


1. Zaskar range
2. Karakoram range
3. Pir Panjal range
4. Ladakh range
Select the correct answer using the code given below:
A. 4-2-1-3
B. 2-4-3-1
C. 4-2-3-1
D. 2-4-1-3
Answer: D
Explanation:
Hence, option D is correct.

2nd Floor, 45 Pusa Road, Opp. Metro Pillar 128, Karol Bagh, New Delhi-110005
Ph: 08045248491, 07041021151 | Email: students@levelupias.com
Q47. Consider the following statements:
Statement-I:
India grows only short staple (Indian) cotton, whereas long staple (American) cotton is grown in Pakistan and Afghanistan.
Statement-II:
India lost a large proportion of cotton growing area to Pakistan during partition.
Which one of the following is correct in respect of the above statements?
A. Both Statement-I and Statement-II are correct and Statement-II is the correct explanation for Statement-I.
B. Both Statement-I and Statement-II are correct and Statement-II is not the correct explanation for Statement-I.
C. Statement-I is correct but Statement-II is incorrect.
D. Statement-I is incorrect but Statement-II is correct.
Answer: D
Explanation:
• Statement-I is not correct: Cotton is a tropical crop grown in kharif season in semi-arid areas of the country. India
grows both short staple (Indian) cotton as well as long staple (American) cotton called ‘narma’ in north-western
parts of the country. India ranks second in the world in the production of cotton after China. Cotton occupies about
4.7 per cent of total cropped area in the country.
o There are three cotton growing areas, i.e. parts of Punjab, Haryana and northern Rajasthan in north-west,
Gujarat and Maharashtra in the west and plateaus of Andhra Pradesh, Karnataka and Tamil Nadu in south.
• Statement-II is correct: India lost a large proportion of cotton growing area to Pakistan during partition. However,
its acreage has increased considerably during the last 75 years. Now, Cotton occupies about 4.7 per cent of the total
cropped area in the country.
Hence, option D is correct.

Q48. Consider the following statements with reference to the Indus River system:
1. Chenab is the largest tributary of the Indus.
2. Jhelum meets Sutlej near Harike Wetland.
3. Ravi flows through Srinagar and passes through Wular lake.
How many of the above statements is/are correct?
A. Only one
B. Only two
C. All three
D. None
Answer: A
Explanation:
• Statement 1 is correct: The Chenab is the largest tributary of the Indus. It is formed by two streams, the Chandra
and the Bhaga, which join at Tandi near Keylong in Himachal Pradesh. Hence, it is also known as Chandrabhaga.
• Statement 2 is not correct: It is Beas (not Jhelum), which originates from the Beas Kund near the Rohtang Pass, meets
the Satluj near Harike Wetland.
• Statement 3 is not correct: The Jhelum rises from a spring at Verinag situated at the foot of the Pir Panjal in the
south-eastern part of the valley of Kashmir. It flows through Srinagar and the Wular lake before entering Pakistan
through a deep narrow gorge. It joins the Chenab near Jhang in Pakistan.
o Ravi rises west of the Rohtang pass in the kullu hills of Himachal Pradesh.
Hence, option A is correct.

2nd Floor, 45 Pusa Road, Opp. Metro Pillar 128, Karol Bagh, New Delhi-110005
Ph: 08045248491, 07041021151 | Email: students@levelupias.com
Q49. Which among the following steps can be undertaken to protect cultivable lands in arid and semi-arid plains of India?
1. Construction of shelter belts of trees
2. Agro-forestry
3. Terrace farming
Select the correct answer using the code given below.
A. 1 only
B. 2 and 3 only
C. 1 and 2 only
D. 1, 2 and 3
Answer: C
Explanation:
• Statement 1 is correct: Construction of shelter belts of trees involves planting trees in rows or belts around
cultivable lands to act as barriers against sand dune encroachment. These trees help stabilize the soil, reduce wind
erosion, and provide shade and windbreaks for crops.
• Statement 2 is correct: Agroforestry involves integrating trees, crops, and/or livestock on the same area of land. In
arid and semi-arid areas, agroforestry practices can help improve soil fertility, water retention, and overall land
productivity. Trees can provide shade and shelter for crops, reduce evaporation, and contribute organic matter to the
soil.
• Statement 3 is not correct: While terrace farming is an important method used to prevent soil erosion and improve
agriculture productivity, it is not typically employed in plain areas. Terrace farming is more commonly used in hilly
or sloped terrain to create flat, level surfaces for cultivation by carving terraces into the hillsides.
Hence, option C is correct.

Q50. Consider the following statements:


1. Cold currents are usually found on the west coast of the continents in the low and middle latitudes.
2. Canary current causes a desiccating effect on the Namib desert.
3. Agulhas current which flows down the African coast is a cold current.
How many of the above statements is/are correct?
A. Only one
B. Only two
C. All three
D. None
Answer: A
Explanation:
• Statement 1 is correct: Cold currents are usually found on the west coast of the continents in the low and middle
latitudes (both hemispheres). The trade winds in the tropical region (low and middle latitudes), blowing from east
to west, generate a westward movement of surface ocean water. This accumulation of water in the western parts of
ocean basins creates a pressure difference, causing upwelling of cold, nutrient-rich water along the western coasts
of continents. This upwelling gives rise to "cold currents" such as the California Current and the Peru Current,
influencing marine ecosystems and coastal climates in the process.
• Statement 2 is not correct: The Canary Current causes a desiccating effect on the Sahara Desert (not on the Namib
desert). The current flows along the western coast of Africa, bringing cool water from the North Atlantic Ocean
southward. This cool water cools the air above it, creating stable atmospheric conditions that inhibit cloud formation
and precipitation. As a result, the region influenced by the Canary Current, including the Sahara Desert, experiences
dry, arid conditions.
2nd Floor, 45 Pusa Road, Opp. Metro Pillar 128, Karol Bagh, New Delhi-110005
Ph: 08045248491, 07041021151 | Email: students@levelupias.com
• Statement 3 is not correct: Agulhas current is a warm current which flows down the east coast of the African
continent. It carries warm water from the Indian Ocean southward along the African coast.
Hence, option A is correct.

Q51. Which of the following statements best describes the term Agro-forestry?
A. Raising and management of trees on privately owned lands around commercial green belts.
B. Raising of trees and agriculture crops on the same land inclusive of the waste patches.
C. Planting of trees on the sides of roads, canals and railways.
D. A type of farming which involves both the growing of crops as well as the raising of livestock.
Answer: B
Explanation:
Agro-forestry:
• It is a sustainable management for the land that increases overall production, combines agricultural crops, tree
crops, forest plants and animals simultaneously and applies management practices that are compatible with the
cultural patterns of the local population.
• It is a type of social forestry in which tree farming and fodder plants, grasses and legumes are grown on the farmers’
land.
• It in order to develop suitable systems of land management involves raising of trees and agricultural crops either
on the same land or in close association in such a way that all land including the waste patches is put to good use.
• The Indian Council for Agricultural Research and Forestry Department jointly undertake agroforestry research which
involves the integration of silviculture, with horticulture, agriculture and animal husbandry.
Hence, option B is correct.

Q52. Consider the following statements with reference to Shola Forests:


1. They are evergreen stunted tropical montane forest.
2. Isolated patches found in the valleys, separated by grasslands.
3. They are found in higher altitude of the Western ghats in Tamil Nadu and Kerala.
How many of the above statements is/are NOT correct?
A. Only one
B. Only two
C. All three
D. None
Answer: D
Explanation:
Shola forest is a tropical montane forest found in the Western Ghats.
• Statement 1 is correct: Shola forests are evergreen forests found in the montane (mountainous) regions. The term
"stunted" refers to the relatively short stature of the trees in these forests due to various environmental factors.
• Statement 2 is correct: Shola forests are often characterized by their isolated patches in valleys, and they are typically
surrounded by grasslands. This mosaic pattern, known as the shola-grassland complex, is a defining feature of these
regions.
• Statement 3 is correct: Shola forests are primarily found in the southern Western Ghats Mountain range, spanning
across the states of Tamil Nadu, Karnataka and Kerala. They typically occur at elevations above 1600 meters, thriving
in the cool, misty environment.
Hence, option D is correct.

2nd Floor, 45 Pusa Road, Opp. Metro Pillar 128, Karol Bagh, New Delhi-110005
Ph: 08045248491, 07041021151 | Email: students@levelupias.com
Q53. Consider the following statements with reference to "Inversion of Temperature":
1. A long winter night with clear skies and still air is ideal situation for inversion.
2. In polar regions, it is a normal phenomenon throughout the year.
Which of the statements given above is/ are correct?
A. 1 only
B. 2 only
C. Both 1 and 2
D. Neither 1 nor 2
Answer: C
Explanation:
Temperature inversion, also called thermal inversion, a reversal of the normal behaviour of temperature in the troposphere
(the region of the atmosphere nearest Earth’s surface), in which a layer of cool air at the surface is overlain by a layer of
warmer air. (Under normal conditions air temperature usually decreases with height.)
• Statement 1 is correct: A long winter night with clear skies and still air is ideal for the development of temperature
inversion. During such conditions, the Earth's surface loses heat rapidly, cooling the air near the surface. If the air is
calm and the skies are clear, there is little vertical mixing of the atmosphere, allowing the formation of an inversion
layer where the temperature increases with height.
• Statement 2 is correct: Temperature inversion in the high Arctic is caused by the lack of surface heating by the Sun,
which stays below the horizon during the winter (the Polar Night), and the continuous loss of heat from the surface
(through emission of infrared radiation). Hence, temperature inversion is common phenomenon in polar regions.
Hence, option C is correct.

Q54. With reference to India, ‘Bewar’, ‘Podu’, ‘Bringa’, ‘Khil’, ‘Kuruwa’ are the names of which of the following?
A. Saline Lakes
B. Slash and Burn Agriculture
C. Type of Millets
D. Artificial Wetlands created by tribals
Answer: B
Explanation:
• The ‘slash and burn’ agriculture is known as ‘Milpa’ in Mexico and Central America, ‘Conuco’ in Venzuela, ‘Roca’ in
Brazil, ‘Masole’ in Central Africa, ‘Ladang’ in Indonesia, ‘Ray’ in Vietnam.
• In India, this primitive form of cultivation is called ‘Bewar’ or ‘Dahiya’ in Madhya Pradesh, ‘Podu’ or ‘Penda’ in Andhra
Pradesh, ‘Pama Dabi’ or ‘Koman’ or Bringa’ in Odisha, ‘Kumari’ in Western Ghats, ‘Valre’ or ‘Waltre’ in South-eastern
Rajasthan, ‘Khil’ in the Himalayan belt, ‘Kuruwa’ in Jharkhand, and ‘Jhumming’ in the North-eastern region.
Hence, option B is correct.

Q55. With reference to type of resources on the basis of the ‘Status of development’, consider the following statements:
1. Resources which are found in a region, but have not been utilized are known as Stock.
2. Resources which can be put into use with the help of existing technical ‘know-how’ but their use has not been started are
known as Reserves.
3. Resources in the environment which have the potential to satisfy human needs but human beings do not have the
appropriate technology to access these are called as Potential resources.
How many of the above statements is/are correct?
A. Only one
B. Only two
2nd Floor, 45 Pusa Road, Opp. Metro Pillar 128, Karol Bagh, New Delhi-110005
Ph: 08045248491, 07041021151 | Email: students@levelupias.com
C. All three
D. None
Answer: A
Explanation:
• Statement 1 is not correct: Resources which are found in a region, but have not been utilized are known as Potential
Resources. For example, the western parts of India particularly Rajasthan and Gujarat have enormous potential for
the development of wind and solar energy, but so far these have not been developed properly.
• Statement 2 is correct: Reserves are the subset of the stock, which can be put into use with the help of existing
technical ‘know-how’ but their use has not been started. These can be used for meeting future requirements. River
water can be used for generating hydroelectric power but presently, it is being utilized only to a limited extent. Thus,
the water in the dams, forests etc. is a reserve which can be used in the future.
• Statement 3 is not correct: Resources in the environment which have the potential to satisfy human needs but
human beings do not have the appropriate technology to access these, are included among stock. For example,
water is a compound of two gases; hydrogen and oxygen. Hydrogen can be used as a rich source of energy. But we do
not have advanced technical ‘know-how’ to use it for this purpose. Hence, it can be considered as stock.
Hence, option A is correct.

Q56. Which of the following statements are correct regarding Footloose Industries?
1. These industries are generally not polluting.
2. Factors like resources or transport have very little effect on costs of these industries.
3. Cement manufacturing is a foot loose industry.
Select the correct answer using the code given below:
A. 1 and 2 only
B. 1, 2 and 3
C. 2 and 3 only
D. 1 and 3 only
Answer: A
Explanation:
• Statement 1 is correct: Footloose industries produce in relatively small quantities, employing smaller workforces
and are considered to be more efficient from an ecological point of view. These are generally not polluting industries.
o For example - Watch-making, Diamond cutting, Precision electronics etc.
• Statement 2 is correct: These industries can be placed and located at any location without effect from factors such
as resources or transport. Their products are having very high value addition and smaller in size and so transportation
and other cost are only a small fraction of total cost.
• Statement 3 is not correct: Cement manufacturing is not a foot loose industry as this is a raw material specific
industry. The final product (cement) weighs significantly more than the raw materials. Transportation costs play a
major role in the industry's location decisions, making it tethered to sources of raw materials like limestone and
clay. Additionally, cement plants require heavy infrastructure and generate significant dust pollution, further limiting
their locational flexibility.
Hence, option A is correct.

Q57. What are ‘Black smokers’, in context of the Earth’s geomorphology?


A. It refers to the hydrothermal vents emitting sulphide rich magma, which turns water black.
B. It refers to the clastic magma emitted by the volcanoes, which turn the skyline black.
C. It refers to the coral which has turned black due to the death of corals.
2nd Floor, 45 Pusa Road, Opp. Metro Pillar 128, Karol Bagh, New Delhi-110005
Ph: 08045248491, 07041021151 | Email: students@levelupias.com
D. It refers to the blackening of air due to the inversion of temperature.
Answer: A
Explanation:
• Black smokers are the hydrothermal vents, where hot water from within the rock jets out into the cold seawater.
These are also known as underwater geysers emitting jets of hot, mineral-rich water.
• As the hot water cools, the metals, sulphur and other elements, dissolved in the hot water, form small crystals
(chimney-like structures) that make the water black and cloudy.
• ‘White smokers’ are chimneys formed from deposits of barium, calcium, and silicon, which are white.
Hence, option A is correct.

Q58. Consider the following statements with reference to Fertigation:


1. It is the application of fertilizer along with irrigation water.
2. It does not work well with water-soluble fertilizers.
3. It is mainly used to replace the traditional irrigation systems in high rainfall areas.
How many of the above statements is/are correct?
A. Only one
B. Only two
C. All three
D. None
Answer: A
Explanation:
• Statement 1 is correct: Fertigation is the process of applying fertilizers to crops through irrigation water. This method
allows for precise control over the application of nutrients, as the fertilizers are delivered directly to the root zone
along with the water.
• Statement 2 is not correct: Fertigation works very well with water-soluble fertilizers. In fact, water-soluble fertilizers
are commonly used in fertigation systems because they can be easily mixed with irrigation water and delivered
efficiently to the plants' root systems.
• Statement 3 is not correct: While fertigation is beneficial for increasing water and nutrient use efficiency, it has
limitations, particularly in high rainfall areas or regions with well-developed traditional irrigation systems.
o In such areas, the use of fertigation may not be as practical or necessary compared to areas where traditional
irrigation methods are less efficient. While fertigation can be used alongside traditional irrigation methods,
it is not mainly used to replace them, especially in areas with high rainfall.
Hence, option A is correct.

Q59. Panna, Para, Palli, Nagla, Dhani are the local names of which types of settlement?
A. Dispersed Settlements
B. Semi-Clustered Settlements
C. Clustered Settlements
D. Hamleted Settlements
Answer: D
Explanation:
• Hamleted Settlements: Sometimes settlements are fragmented into several units physically separated from each
other bearing a common name. These units are locally called panna, para, palli, nagla, dhani etc. in various parts of
the country. This segmentation of a large village is often motivated by social and ethnic factors. Such villages are
more frequently found in the middle and lower Ganga plain, Chhattisgarh and lower valleys of the Himalayas.
2nd Floor, 45 Pusa Road, Opp. Metro Pillar 128, Karol Bagh, New Delhi-110005
Ph: 08045248491, 07041021151 | Email: students@levelupias.com
Hence, option D is correct.

Additional Info:
• Clustered, agglomerated or nucleated: The clustered rural settlement is a compact or closely built-up area of houses.
In this type of village, the general living area is distinct and separated from the surrounding farms, barns and pastures.
The closely built-up area and its intervening streets present some recognisable pattern or geometric shape, such as
rectangular, radial, linear, etc. Such settlements are generally found in fertile alluvial plains and in the northeastern
states.
• Semi-clustered or fragmented: Semi-clustered or fragmented settlements may result from a tendency of clustering in
a restricted area of dispersed settlement. More often such a pattern may also result from segregation or
fragmentation of a large compact village.
• Dispersed or isolated: Dispersed or isolated settlement pattern in India appears in the form of isolated huts or
hamlets of few huts in remote jungles, or on small hills with farms or pasture on the slopes. Extreme dispersion of
settlement is often caused by the extremely fragmented nature of the terrain and land resource base of habitable
areas.

Q60. Consider the following statements with reference to Steel production in India:
1. In FY23, India is the 2nd largest producer of crude steel in the world.
2. In FY23, India is the 2nd largest consumer of finished steel in the world.
3. Jharkhand and Chhattisgarh are important steel-producing states in India.
How many of the above statements is/are correct?
A. Only one
B. Only two
C. All three
D. None
Answer: C
Explanation:
• Statement 1 is correct: India is the world's 2nd largest producer of crude steel, with an output of 125.32 million
tonnes (MT) of crude steel and 121.29 MT of finished steel production in FY23.
o The steel industry in India has experienced substantial growth in the past decade, with a 75% increase in
production since 2008.
• Statement 2 is correct: India has become the 2nd largest consumer of finished steel in the world. The finished steel
consumption was 106 million tons (MT) during the FY22.
o The per capita steel consumption in the World is around 233 kg. However, the per capita consumption of steel
in India is around 77.2 kg, gone up by 50% in last 8 years, which is 1/3rd of the average world per capita steel
consumption.
• Statement 3 is correct: More than 80% of the country’s reserves are in the states of Odisha, Jharkhand, West Bengal,
Chhattisgarh and the northern regions of Andhra Pradesh.
o Important steel-producing centers - Bhilai (Chhattisgarh), Durgapur (West Bengal), Burnpur (West Bengal),
Jamshedpur (Jharkhand), Rourkela (Odisha), Bokaro (Jharkhand).
Hence, option C is correct.

Q61. Consider the following statements with reference to the “Aravalli Range”:
1. It is one of the oldest mountain ranges of the world and predates the formation of the Himalayan ranges.
2. The highest peak is Guru Shikhar on Mount Abu.
2nd Floor, 45 Pusa Road, Opp. Metro Pillar 128, Karol Bagh, New Delhi-110005
Ph: 08045248491, 07041021151 | Email: students@levelupias.com
3. It is devoid of any type of mineral deposits.
4. Luni River originates from this range.
5. The hidden limb of the Aravallis creates a divide between the drainage of rivers of the Ganga and the Indus.
How many of the above statements are correct?
A. Only two
B. Only three
C. Only four
D. All five
Answer: C
Explanation:
• Statement 1 is correct: The Aravali Mountain ranges in India is one of the oldest fold mountains in the world. The It
dates back to 3.2 to 1.2 billion years ago, of shoving and pushing of tectonic plates and magma outpourings, while
Himalayas were formed when India rammed into Asia about 40 to 50 million years ago.
• Statement 2 is correct: The highest peak is Guru Shikhar at 1,722 metres (5,650 ft) in the Arbuda Mountains of
Rajasthan.
• Statement 3 is not correct: The Aravalli Range is rich in mineral deposits, including copper, zinc, lead, and marble.
• Statement 4 is correct: Luni River originates in the Pushkar valley of the Aravalli Range, near Ajmer, and passes
through the southeastern portion of the Thar Desert.
• Statement 5 is correct: The hidden limb of the Aravallis that extends from Delhi to Haridwar creates a divide
between the drainage of rivers of the Ganga and the Indus.
Hence, option C is correct.

Q62. Consider the following statements with reference to Soil profile and horizons:
1. There is a gradual increase of organic matter with increasing depth.
2. The level of aeration of the soil decreases from the surface downwards in the soil profile.
Which of the statements given above is/are correct?
A. 1 only
B. 2 only
C. Both 1 and 2
D. Neither 1 nor 2
Answer: B
Explanation:
Soil profile is defined as the vertical section of the soil from the ground surface downwards to where the soil meets the
underlying rock. Soil horizons are the layers in the soil as you move down the soil profile.
• Statement 1 is not correct: On average, there is gradual decrease of organic matter, number of living organisms,
activity of living organisms, etc. with increasing depth in the soil profile. This is because organic materials like
decomposed plant and animal matter tend to accumulate in the topsoil (the uppermost layer of soil), contributing to
its fertility. As you move deeper into the soil profile, the decomposition process and microbial activity generally
decrease, leading to lower organic matter content.
• Statement 2 is correct: Aeration refers to the presence of air in the soil, which is crucial for the growth of plant roots
and the activity of soil organisms. Near the surface, the soil is typically more porous and has more air-filled spaces,
providing better aeration. As you go deeper into the soil profile, the compaction of soil particles and the reduction
in pore spaces can limit the movement of air, leading to decreased aeration in the lower layers.
Hence, option B is correct.

2nd Floor, 45 Pusa Road, Opp. Metro Pillar 128, Karol Bagh, New Delhi-110005
Ph: 08045248491, 07041021151 | Email: students@levelupias.com
Q63. Consider the following statements with reference to Urea Fertilizer:
1. Urea is an organic compound which can only be synthetically produced.
2. Urea is the only fertilizer in India that is under statutory price control.
3. Urea production in India is reserved for the public sector.
How many of the above statements is/are correct?
A. Only one
B. Only two
C. All three
D. None
Answer: A
Explanation:
• Statement 1 is not correct: Urea can be produced both by natural and synthetic processes. It is naturally produced
by a lot of animals as a metabolic byproduct such as urine and manure and such natural urea is an approved Organic
fertilizer. It can be synthetically produced or manufactured in a factory using natural gas, ammonia gas as major raw
material.
• Statement 2 is correct: Urea is the only fertilizer at present with pricing and distribution being controlled statutorily
by the Government. Thus, no one can sell urea above the MRP declared by the Govt.
• Statement 3 is not correct: Urea production in India is not exclusively reserved for the public sector. Public, Private
and cooperative sectors are involved in the production of urea. While there are government-owned plants, there are
also private companies that produce urea in India. The involvement of these sectors helps meet the demand for urea
in the agricultural sector.
Hence, option A is correct.

Q64. Consider the following statements:


1. Census towns are areas that have more than 5,000 people, a density greater than 400 persons per sq. km, and 75% of the
male working population in non-agricultural activities.
2. Statutory Town is one with a municipality, corporation, cantonment board or notified town area committee.
Which of the statements given above is/are NOT correct?
A. 1 only
B. 2 only
C. Both 1 and 2
D. Neither 1 nor 2
Answer: D
Explanation:
• Statement 1 is correct: Census towns are areas that the census classifies as urban because they have more than 5,000
people, a density greater than 400 persons per sq. km, and 75% of the male working population in non-agricultural
activities. However, census towns are governed by rural local bodies (RLBs) or Panchayats. As per the 2011 Census,
there are 3,784 Census Town in India.
• Statement 2 is correct: Statutory Town is one with a municipality, corporation, cantonment board or notified town
area committee. As per 2011 Census, there are 4,041 such towns in India.
Hence, option D is correct.

Q65. The Population Pyramid of Nation “XYZ” is broader at base and rapidly narrows at the top. The pyramid is also
symmetrical about the vertical axis. Which of the following can be said with certainty about Nation “XYZ”?
1. Sex-ratio is adverse in the adult age group.
2nd Floor, 45 Pusa Road, Opp. Metro Pillar 128, Karol Bagh, New Delhi-110005
Ph: 08045248491, 07041021151 | Email: students@levelupias.com
2. Both Birth rate and death rates are high.
3. Population is geographically concentrated.
Select the correct answer using the code given below:
A. 1 and 2 only
B. 2 only
C. 1 and 3 only
D. 1, 2 and 3
Answer: B
Explanation:
A population pyramid is the graphical representation of the distribution of
the population by sex, and age group. It takes the shape of a pyramid when
the population is growing. Population pyramids are also known as age
pyramids because it is a graphical representation of age.
• Statement 1 is not correct: Since the pyramid is vertically
symmetrical, sex-ratio is 1000, i.e. males and females are equal in
number (not adverse).
• Statement 2 is correct: The population pyramid of a country in
which birth and death rates both are high is broad at the base and
rapidly narrows towards the top. This is because although, many
children are born, a large percentage of them die in their infancy, relatively few become adults and there are very
few old people.
• Statement 3 is not correct: Population pyramid does not tell anything about the geographical distribution of
population.
Hence, option B is correct.

Q66. "This race includes the Europeans and Aryans of the Indian subcontinent. They are fair-skinned people with a long and
narrow nose. They are further divided into different groups which include Nordics of North Europe and Alpine of Central
Europe".
Which of the following races is being described in the above passage?
A. Caucasoids
B. Negroids
C. Australoids
D. Mongoloids
Answer: A
Explanation:
A human race is defined as a group of people with certain common inherited features that distinguish them from other
groups of people. It generally differentiates a group of people on the basis of some visible physical traits, such as skin color,
hair texture, facial features, and eye formation.
• Caucasoids:
o Europe, the Horn of Africa, West Asia, Central Asia, and South Asia.
o Fair to light skin tones.
o Straight to wavy hair.
o Prominent noses, often described as long and narrow.
o Eye color blue/dark brown with various shapes.
Hence, option A is correct.
2nd Floor, 45 Pusa Road, Opp. Metro Pillar 128, Karol Bagh, New Delhi-110005
Ph: 08045248491, 07041021151 | Email: students@levelupias.com
Addition Info:
• Negroids:
o Sub-Saharan and Central African.
o Dark skin tones.
o Coiled or kinky hair.
o Broad noses.
o Full lips.
• Australoids:
o Indigenous people of Australia, Papua New Guinea, and parts of Southeast Asia.
o Variable skin tones.
o Wavy to curly hair.
o Facial features can vary widely.
• Mongoloids:
o East Asian, Central Asian, Native American, and Arctic Indigenous populations.
o Yellowish to reddish-brown skin tones.
o Straight black hair.
o Epicanthic folds (skin folds covering the inner corner of the eye).
o Generally, facial features like a broad and flat face.

Q67. With reference to 'Rare earth metals', consider the following statements:
1. They are a group of metals which are rarely found in the Earth's crust.
2. Extracting these metals is hazardous for health and the environment.
3. China accounts for approximately 70% of the total production of rare earths in the world.
How many of the above statements is/are correct?
A. Only one
B. Only two
C. All three
D. None
Answer: B
Explanation:
o Statement 1 is not correct: Rare earth metals are not rare in terms of abundance in the Earth's crust such as Cerium
is about 15,000 times more abundant than gold. They are called 'rare earths' because, in the past, it was
technologically difficult to extract them from their oxide forms.
o Statement 2 is correct: The mining and processing of rare earth metals can be harmful to health and the
environment. It can involve toxic chemicals, radioactive materials, and generate dust that can cause respiratory
problems. Additionally, the process often requires a lot of water and energy, which can strain local resources.
o Statement 3 is correct: China currently dominates the global rare earth market, accounting for over 70% of the
production. This dominance has raised concerns about potential supply chain disruptions and geopolitical influence.
Hence, option B is correct.

Q68. Consider the following statements:


1. Glaciers form only on land.
2. Around 99% of glacial ice is contained within vast ice sheets in the polar regions.
3. Glacial ice is the largest reservoir of fresh water on Earth.

2nd Floor, 45 Pusa Road, Opp. Metro Pillar 128, Karol Bagh, New Delhi-110005
Ph: 08045248491, 07041021151 | Email: students@levelupias.com
How many of the above statements is/are correct?
A. Only one
B. Only two
C. All three
D. None
Answer: C
Explanation:
• Statement 1 is correct: Glaciers are made up of fallen snow that, over many years, compresses into large, thickened
ice masses. It forms when snow remains in one location long enough to transform into ice. However, they can also
flow and move over land, carving out valleys and shaping landscapes. They exist and move on land (not in water).
• Statement 2 is correct: The majority of glacial ice is found in ice sheets, particularly in Antarctica and Greenland,
which are massive accumulations of ice that cover extensive areas. They make up around 99% of all glacial ice on
Earth.
• Statement 3 is correct: About three-quarters of Earth's freshwater is stored in glaciers. Therefore, glacier ice is the
second largest reservoir of water on Earth and the largest reservoir of freshwater on Earth.
Hence, option C is correct.

Q69. Consider the following statements:


1. More fresh water can be found in the atmosphere than in the rivers on Earth.
2. Rivers on earth contain more fresh water than fresh water lakes.
Which of the statements given above is/are correct?
A. 1 only
B. 2 only
C. Both 1 and 2
D. Neither 1 nor 2
Answer: A
Explanation:
• Statement 1 is correct: The Earth's atmosphere
contains a significant amount of water vapor, and
water in the atmosphere is considered freshwater
which is more than the water in the rivers on Earth.
This water vapor is part of the Earth's hydrological cycle.
• Statement 2 is not correct: Freshwater lakes contain
more water than rivers on Earth. Rivers are generally
smaller in volume compared to lakes. Lakes accumulate
and store water over time, while rivers serve as channels that transport water from higher elevations to lower
elevations.
Hence, option A is correct.

Q70. Consider the following statements:


Statement-I:
Ocean vents are primarily found around mid-ocean ridges and volcanic arcs.
Statement-II:
Ocean vents are the product of tectonic activity beneath the ocean floor.
Which one of the following is correct in respect of the above statements?
2nd Floor, 45 Pusa Road, Opp. Metro Pillar 128, Karol Bagh, New Delhi-110005
Ph: 08045248491, 07041021151 | Email: students@levelupias.com
A. Both Statement-I and Statement-II are correct and Statement-II is the correct explanation for Statement-I.
B. Both Statement-I and Statement-II are correct and Statement-II is not the correct explanation for Statement-I.
C. Statement-I is correct but Statement-II is incorrect.
D. Statement-I is incorrect but Statement-II is correct.
Answer: A
Explanation:
Ocean vents are a type of hydrothermal vent. Other types of hydrothermal vents include hot springs, geysers, and fumaroles.
As their name indicates, all hydrothermal vents are characterized by water (hydro-) and extremely high temperatures
(thermal).
• Statement-I is correct: Ocean vents are primarily found around mid-ocean ridges and volcanic arcs. At both mid-
ocean ridges and back arc basins, the molten magma of Earth's asthenosphere wells up close to the surface.
• Statement-II is correct: The movement and interaction of tectonic plates are the fundamental forces behind the
creation of ocean vents. Tectonic activity opens fissures in the ocean crust, enabling seawater circulation and its
subsequent heating by magma.
o Ocean vents are found in all ocean basins, although they are most abundant around the Pacific Ocean’s “Ring
of Fire,” which also includes active earthquake zones, volcanoes, and ocean trenches.
Hence, option A is correct.

Q71. Consider the following statements:


1. Bagar is the semi-desert area to the west of Aravallis range.
2. Underlying rock structure of Thar desert is an extension of the Peninsular plateau in India.
3. ‘Kayals’ are backwaters along the Eastern coast of India.
4. Karewas are the thick deposits of glacial clay and other materials embedded with moraines.
How many of the above statements is/are correct?
A. Only one
B. Only two
C. Only three
D. All four
Answer: C
Explanation:
• Statement 1 is correct: Bagar refers to the semi-desert area which is west of Aravallis. Bagar has a thin layer of sand.
It is drained by Luni in the south whereas the northern section has a number of salt lakes.
• Statement 2 is correct: Underlying rock structure of the Indian desert is an extension of the Peninsular plateau. Due
to extreme arid conditions, its surface features have been carved by physical weathering and wind actions. Some of
the well pronounced desert land features present here are mushroom rocks, shifting dunes and oasis (mostly in its
southern part).
• Statement 3 is not correct: The rivers flowing through western coastal plain do not form any delta. The Malabar coast
has got certain distinguishing features in the form of ‘Kayals’ (backwaters), which are used for fishing, inland
navigation and also due to its special attraction for tourists. ‘Kayals’ are backwaters along the western coast of India.
• Statement 4 is correct: Karewas are the thick deposits of glacial clay and other materials embedded with moraines.
The Kashmir Himalayas are also famous for Karewa formations, which are useful for the cultivation of Zafran, a local
variety of saffron.
Hence, option C is correct.

2nd Floor, 45 Pusa Road, Opp. Metro Pillar 128, Karol Bagh, New Delhi-110005
Ph: 08045248491, 07041021151 | Email: students@levelupias.com
Q72. Consider the following pairs:
Mineral Mines in India
1. Gold Hutti
2. Manganese Sundergarh
3. Silver Zawwar
4. Iron Khetri
5. Copper Balaghat
How many of the above pairs are correct?
A. Only two
B. Only three
C. Only four
D. All five
Answer: C
Explanation:
• Pair 1 is correct: The famous gold mines in India are Kolar (Karnataka), Hutti (Karnataka), Ramagiri (Andhra Pradesh),
and Lawa (Jharkhand).
• Pair 2 is correct: The five largest silver mines by production in India are Sindeshar Khurd (Rajasthan), Rampura
Agucha Mine (Rajasthan), Rajpura Dariba Mine (Rajasthan) Zawwar (Rajasthan), Hutti (Karnataka).
• Pair 3 is correct: Manganese is an important raw material for smelting of iron ore and also used for manufacturing
ferro alloys. Major mines in Odisha are located in the central part of the iron ore belt of India, particularly in Bonai,
Kendujhar, Sundergarh, Gangpur, Koraput, Kalahandi and Bolangir.
• Pair 4 is not correct: About 95 per cent of total reserves of iron ore is located in the States of Odisha, Jharkhand,
Chhattisgarh, Karnataka, Goa, Telangana, Andhra Pradesh and Tamil Nadu. In Odisha, iron ore occurs in a series of hill
ranges in Sundergarh, Mayurbhanj and Jhar. The important mines are Gurumahisani, Sulaipet, Badampahar
(Mayurbhaj), Kiruburu (Kendujhar) and Bonai (Sundergarh). Khetri mines are famous for copper (not for iron).
• Pair 5 is correct: The Copper deposits mainly occur in Singhbhum district in Jharkhand, Balaghat district in Madhya
Pradesh and Jhunjhunu and Alwar districts in Rajasthan.
Hence, option C is correct.

Q73. In which of the following regions of India are shale gas resources found?
1. Cambay Basin
2. Cauvery Basin
3. Krishna-Godavari Basin
4. Assam-Arakan Basin
5. Vindhya Basin
Select the correct answer using the code given below:
A. 1, 2 and 3 only
B. 1, 2 and 4 only
C. 3 and 5 only
D. 1, 2, 3, 4 and 5
Answer: D
Explanation:
• Oil Shales are usually fine-grained sedimentary rocks containing relatively large amounts of organic matter from
which significant quantities of shale oil and combustible gas can be extracted by destructive distillation.

2nd Floor, 45 Pusa Road, Opp. Metro Pillar 128, Karol Bagh, New Delhi-110005
Ph: 08045248491, 07041021151 | Email: students@levelupias.com
• Shale gas can emerge as an important new source of energy in the country. India has several Shale Formations which
seem to hold shale gas.
• The Shale Gas Formations are spread over several sedimentary basins, such as, Gangetic plain, Gujarat, Rajasthan,
Andhra Pradesh and other coastal areas in the country including hydrocarbon-bearing ones - Cambay, Assam Arkan
& Damodar Basins have large shale deposits. The recoverable reserves are identified in Pranhita-Godavari, Damodar
valley and Vindhya Basins.
Hence, option D is correct.

Q74. Consider the following statements with reference to Atal Tunnel:


1. It is built by National Highways Authority of India (NHAI).
2. It is built with ultra-modern specifications in the Dhauladhar range of Himalayas.
3. It connects Manali to Lahaul-Spiti valley throughout the year.
How many of the statements is/are correct?
A. Only one
B. Only two
C. All three
D. None
Answer: A
Explanation:
• Statement 1 is not correct: The World’s longest Highway tunnel, Atal Tunnel (9.02 Km) has been built by Border Road
Organisation.
• Statement 2 is not correct: The Tunnel is built with ultra-modern specifications in the Pir Panjal range of Himalayas
at an altitude of 3000 metres from the Mean Sea Level (MSL). The tunnel, besides offering the Armed Forces a strategic
advantage by providing an alternate link to the critical Ladakh Sector, has also been a boon for the residents of Lahaul
& Spiti District in Himachal Pradesh.
• Statement 3 is correct: This tunnel connects Manali to Lahaul-Spiti valley throughout the year. Earlier the valley was
cut off for about 6 months each year owing to heavy snowfall.
Hence, option A is correct.

Q75. Consider the following statements with reference to Ports in India:


1. Kandla Port is situated at the head of Gulf of Cambay to reduce the pressure at Mumbai port.
2. Marmagao Port is situated at the mouth of the Mahadayi (Mandovi) river, is a natural harbour in Goa.
Which of the above statements is/are correct?
A. 1 only
B. 2 only
C. Both 1 and 2
D. Neither 1 nor 2
Answer: D
Explanation:

2nd Floor, 45 Pusa Road, Opp. Metro Pillar 128, Karol Bagh, New Delhi-110005
Ph: 08045248491, 07041021151 | Email: students@levelupias.com
• Statement 1 is not correct: Kandla Port situated at the
head of Gulf of Kutch has been developed as a major port
to cater to the needs of western and north western parts of
the country and also to reduce the pressure at Mumbai
port. The port is specially designed to receive large
quantities of petroleum and petroleum products and
fertiliser.

• Statement 2 is not correct: Marmagao Port, situated at the


entrance of the Zuari estuary, is a natural harbour in Goa.
It gained significance after its remodeling in 1961 to handle
iron-ore exports to Japan. Construction of Konkan railway
has considerably extended the hinterland of this port.
Karnataka, Goa, Southern Maharashtra constitute its
hinterland.
Hence, option D is correct.

Q76. Consider the following statements:


1. Spring tides occur when the sun, the moon and the earth are in a straight line.
2. Neap tides occur when the sun and moon are at right angles to each other relative to Earth.
Which of the statements given above is/are correct?
A. 1 only
B. 2 only
C. Both 1 and 2
D. Neither 1 nor 2
Answer: C
Explanation:
• Statement 1 is correct: Spring tides occur when the Sun, Moon, and Earth are aligned, meaning they are either all in
a straight line (during full moon) or on opposite sides of Earth (during new moon).
o In these configurations, the gravitational forces of the Sun and Moon combine to create the largest tidal
bulges, resulting in higher high tides and lower low tides.
• Statement 2 is correct: Neap tides occur when the Sun and Moon are at right angles to each other relative to Earth
(during first and third quarter moons).
o In this case, the gravitational forces of the Sun and Moon partially cancel each other out, leading to smaller
tidal bulges and consequently, lower high tides and higher low tides compared to spring tides.
Hence, option C is correct.

Q77. Consider the following statements with reference to the variations in temperature of ocean waters:
1. The annual range of temperature of ocean waters is larger than the temperature of land.
2. Northern hemispherical waters are cooler than southern hemispherical waters.
3. Enclosed seas in low latitudes are relatively cooler than the open seas.
How many of the above statements is/are correct?
A. Only one
B. Only two
C. All three
2nd Floor, 45 Pusa Road, Opp. Metro Pillar 128, Karol Bagh, New Delhi-110005
Ph: 08045248491, 07041021151 | Email: students@levelupias.com
D. None
Answer: D
Explanation:
• Statement 1 is not correct: Water has a higher specific heat capacity compared to land. It takes more energy to raise
the temperature of water than the temperature of an equivalent amount of land. Water warms up and cools down
much more slowly than the land, the annual range of temperature in any part of the ocean is very small. Land heats
up and cools down quickly, leading to larger temperature variations throughout the year.
• Statement 2 is not correct: The Northern Hemisphere has a higher percentage of landmass compared to the
Southern Hemisphere. As a result, northern hemisphere receives more heat due to their contact with a larger extent
of land than the oceans in the southern hemisphere. Hence Northern hemispherical waters are warmer than
southern hemispherical waters.
• Statement 3 is not correct: Enclosed seas in low latitudes, such as the Mediterranean Sea, are often warmer than
open seas in the same latitudes. The limited water exchange in enclosed seas, along with factors like shallower depths
and more exposure to direct sunlight, can contribute to higher temperatures. Open seas, with more significant water
volumes and greater depth, tend to have more temperature stability.
o On the other hand, enclosed seas in the high latitudes have a lower temperature than the open seas.
Hence, option D is correct.

Q78. Arrange the following mountain hills from North to South direction:
1. Javadi hills
2. Palkonda hills
3. Shevaroy hills
4. Garhjat hills
5. Nallamala hills
Select the correct answer using the code given
below:
A. 5-4-2-1-3
B. 5-4-3-1-2
C. 4-5-1-2-3
D. 4-5-2-1-3
Answer: D
Explanation:
Correct order (North to South):
• Garhjat hills
• Nallamala hills
• Palkonda hills
• Javadi hills
• Shevaroy hills
Hence, option D is correct.

2nd Floor, 45 Pusa Road, Opp. Metro Pillar 128, Karol Bagh, New Delhi-110005
Ph: 08045248491, 07041021151 | Email: students@levelupias.com
Q79. Consider the following statements:
1. The retreating south-west monsoon season is marked by clear skies and rise in temperature.
2. ‘October Heat’ is related to the retreating south-west monsoon.
3. The weather in the retreating monsoon is dry in north India, but it is associated with rain in the eastern part of the
Peninsula.
How many of the above statements is/are correct?
A. Only one
B. Only two
C. All three
D. None
Answer: C
Explanation:
• Statement 1 and 2 are correct: The months of October and November are known for retreating monsoons. As the
monsoon winds retreat, they pull moisture away, leading to clearer skies and a gradual increase in temperatures
across North India. This is particularly noticeable in October, giving rise to the phenomenon known as "October
Heat." The combination of reduced cloud cover and high humidity due to the still-moist landmass traps heat, creating
a muggy and uncomfortable feeling.
• Statement 3 is correct: While North India experiences drier weather during the retreating monsoon, the eastern
coast of the Peninsula, including Tamil Nadu and Andhra Pradesh, receives rainfall from the northeast monsoon
winds blowing from the Bay of Bengal. This is why these regions often see their peak rainfall months in October and
November.
Hence, option C is correct.

Q80. Consider the following statements with reference to Coffee:


1. India produces more Robusta coffee, as compared to Arabica coffee.
2. Karnataka is the largest producer of coffee in India.
3. India is a net exporter of coffee.
How many of the above statements is/are NOT correct?
A. Only one
B. Only two
C. All three
D. None
Answer: D
Explanation:
• Statement 1 is correct: India is one of the world's largest producers of Robusta coffee, accounting for around 70%
of its total coffee production.
o Arabica coffee, known for its milder flavour and aroma, makes up the remaining 30%.
• Statement 2 is correct: Karnataka, contributes a whopping 71% to the country's coffee production. The hill ranges
of Kodagu within Karnataka are particularly famous for their coffee estates.
• Statement 3 is correct: India is a net exporter of coffee. Approximately 65% to 70% of the coffee produced in India
is exported and remaining coffee is consumed domestically. Italy, Germany and Russia are the major export
destinations for the Indian coffee.
Hence, option D is correct.

2nd Floor, 45 Pusa Road, Opp. Metro Pillar 128, Karol Bagh, New Delhi-110005
Ph: 08045248491, 07041021151 | Email: students@levelupias.com
Q81. Arrange the following cities in the declining trend of rainfall during the south-west monsoon period in India:
1. Delhi
2. Allahabad
3. Kolkata
4. Patna
Select the correct sequence using the code given below:
A. 3-4-1-2
B. 4-3-1-2
C. 3-4-2-1
D. 4-3-2-1
Answer: C
Explanation:
• Kolkata typically receives a substantial amount of rainfall (119 cm) during the south-west monsoon due to its location
in the eastern part of India, close to the Bay of Bengal.
• Patna, located in the northern part of India, also receives a considerable amount of rainfall (105 cm) during this
period, although it may be slightly lower than in Kolkata.
• Allahabad, located further inland compared to Kolkata and Patna, tends to receive less rainfall (76 cm) during the
south-west monsoon period.
• Delhi, being the furthest inland among the cities mentioned, generally receives the least amount of rainfall (56 cm)
during the south-west monsoon period compared to the other cities.
Hence, option C is correct.

Q82. Consider the following statements with reference to the Chhota Nagpur Plateau:
1. It covers much of Jharkhand state as well as adjacent parts of Odisha, West Bengal, Bihar and Chhattisgarh.
2. Large reserves of iron, coal and manganese are found in this plateau.
3. A complete absence of waterfalls or dams is a distinctive feature of this plateau.
4. Sal forests predominate the plateau.
How many of the above statements is/are correct?
A. Only one
B. Only two
C. Only three
D. All four
Answer: C
Explanation:
• Statement 1 is correct: The plateau covers much of Jharkhand state as well as adjacent parts of Odisha, West Bengal,
Bihar and Chhattisgarh. The Indo-Gangetic plain lies to the north and east of the plateau, and the basin of the
Mahanadi River lies to the south. The total area of the Chota Nagpur Plateau is approximately 65,000 square
kilometres.
• Statement 2 is correct: Chhota Nagpur plateau is a store house of mineral resources such as mica, bauxite, copper,
limestone, iron ore and coal. The Damodar valley is rich in coal and it is considered as the prime centre of coking
coal in the country. Massive coal deposits are found in the central basin spreading over 2,883 square kilometres. The
important coalfields in the basin are Jharia, Raniganj, Bokaro, Ramgarh, Karanpura.
• Statement 3 is not correct: In the plateau areas, there may be several waterfalls as the river falls from a great height,
such as the Hundru Falls, Dassam Falls. Also, some famous dams such as Tilaiya, Konar, Maithan, Panchet and
Tenughat Dam.
2nd Floor, 45 Pusa Road, Opp. Metro Pillar 128, Karol Bagh, New Delhi-110005
Ph: 08045248491, 07041021151 | Email: students@levelupias.com
• Statement 4 is correct: The plateau is covered with a variety of various habitats of which Sal Forest is predominant.
The Chhota Nagpur dry deciduous forests, a tropical and subtropical dry broadleaf forests ecoregion, encompasses
the plateau.
Hence, option C is correct.

Q83. The region is characterized by hot, dry summers and cool, wet winters, found mostly on the western margins of
continents. Sirocco and Mistral are two types of winds that are common in these regions. Trees with small broad leaves are
widely spaced and never very tall. The absence of shade is a distinct feature of this region. The region is also known as the
world’s orchard lands. A wide range of citrus fruits are grown.
The above passage corresponds best to which type of climate?
A. Equatorial Climate
B. Savanna Climate
C. Mediterranean climate
D. Steppe Climate
Answer: C
Explanation:
Mediterranean Climate:
• The Mediterranean climate, distinguished by hot, dry
summers and cool, wet winters, is prevalent on the
western edges of continents, such as the Mediterranean
Basin, parts of California, South Australia, Chile, and
South Africa. Characterized by the Sirocco, a warm wind
from the Sahara, and the Mistral, a cold wind from the
continent's interior, this climate supports vegetation
featuring widely spaced, small trees with small broad
leaves, including olive trees and grapevines. The scarcity
of dense vegetation leads to an absence of significant
shade. Renowned as the world's orchard lands,
Mediterranean regions thrive in agriculture, cultivating a
diverse range of citrus fruits alongside olives and grapes.
Hence, option C is correct.

Q84. Consider the following statements with reference to Migration in India as per Census 2011:
1. The intra-state migration is dominated by female migrants as compared to male population.
2. The rural to urban inter-state migration is dominated by male migrants.
3. Tamil Nadu receives the highest number of migrants in India.
4. The largest share of international migrants in India is from Nepal.
How many of the above statements is/are correct?
A. Only one
B. Only two
C. Only three
D. All four
Answer: B

2nd Floor, 45 Pusa Road, Opp. Metro Pillar 128, Karol Bagh, New Delhi-110005
Ph: 08045248491, 07041021151 | Email: students@levelupias.com
Explanation:
• Statement 1 is correct: The intra-state migration is categorized into four types - rural to rural, rural to urban, urban
to urban and urban to rural. As per 2011 Census of India, among the individuals classified as "migrants," merely
11.91% (5.43 crore) had relocated from one state to another, whereas a substantial 39.57 crore had migrated within
their respective states. In all four categories, the majority of these migrants were females. The predominant reason
for migration in most cases was marriage.
• Statement 2 is correct: In case of inter-state migration, economic factors often lead to a predominance of men in
the rural-to-urban stream. Simultaneously, in the context of shorter distances within states, females tend to dominate
the rural-to-rural inter-state migration streams.
• Statement 3 is not correct: According to the Census 2011 data on migration, Maharashtra stands out as the state
with the highest number of migrants in India. Among the 5.74 crore migrants in Maharashtra, 27.55 lakh indicated
Uttar Pradesh as their last place of residence, while 5.68 lakh mentioned Bihar. Internal migration within Maharashtra
accounted for the majority of migrants, with 4.79 crore individuals moving within the state.
• Statement 4 is not correct: As per Census 2011, the largest share of international migrants in India was from
Bangladesh (not Nepal). This reflects the fact that a significant number of people from Bangladesh migrate to India
for various reasons.
Hence, option B is correct.

Q85. Consider the following statements with reference to the Indian Islands:
1. The Andaman is considered to be the extension of the submerged Arakan Yoma Mountain range of Myanmar.
2. Minicoy Island is the largest island of the Lakshadweep.
3. Pamban Island is a rocky island and represents the extension of the peninsular landmass in Tamil Nadu.
How many of the above statements is/are correct?
A. Only one
B. Only two
C. All three
D. None
Answer: B
Explanation:
• Statement 1 is correct: The Andaman Islands, located in the Bay of Bengal, are geologically connected to the
submerged Arakan Yoma Mountain range in Myanmar. They are the result of the interaction of the Indo-Australian
Plate and the Eurasian Plate.
• Statement 2 is not correct: Andrott Island is the largest island of Lakshadweep whereas Minicoy is the second-
largest and southernmost island of this group.
• Statement 3 is correct: Pamban Island is situated between India and Sri Lanka and is a rocky island and represents
the extension of peninsular landmass in the Ramnad district in Tamil Nadu. Fringing coral reefs are very popular
here.
Hence, option B is correct.

Q86. ‘Molasses Basin', made up of soft unconsolidated deposits, is an important physiological feature of which of the following
state?
A. Arunachal Pradesh
B. Sikkim
C. Himachal Pradesh
D. Mizoram
2nd Floor, 45 Pusa Road, Opp. Metro Pillar 128, Karol Bagh, New Delhi-110005
Ph: 08045248491, 07041021151 | Email: students@levelupias.com
Answer: D
Explanation:
Mizoram is referred to as the "Molasses Basin" due to its geological composition featuring unconsolidated sedimentary
deposits. These deposits were formed from eroded materials of the rising mountains flanking the region.
Hence, option D is correct.

Q87. Which of the following best describes the term ‘Warabandi system’?
A. A type of slash and burn agriculture system.
B. A rotational water allocation system.
C. A system to convert wasteland into cultivated land.
D. A system to collect Agricultural waste.
Answer: B
Explanation:
Warabandi System:
• Warabandi is a traditional water distribution system used in various parts of India including Madhya Pradesh,
particularly in regions with canal irrigation. It involves the rotation of water supply among different farmers or fields
in a systematic manner. This system helps in equitable distribution of water resources and ensures that each farmer
gets a fair share of water for irrigation.
Hence, option B is correct.

Q88. Consider the following pairs:


Intrusive Volcanic
Description
Forms
1. Dykes Lava making its way vertically through fissures.
Large body of magmatic material that cools in the
2. Sills deeper depth of the crust molds in the form of
large domes.
Thick horizontal deposits of the intrusive igneous
3. Batholiths
rocks.
How many of the pairs given above is/are correctly matched?
A. Only one
B. Only two
C. All three
D. None
Answer: A
Explanation:
• Pair 1 is correct: Dykes - When the lava makes its way vertically through cracks and the fissures developed in the
land, it solidifies almost perpendicular to the ground. It gets cooled in the same position to develop a wall-like
structure. Such structures are called dykes.
• Pair 2 is not correct: Sills - The near horizontal bodies of the intrusive igneous rocks are called sill. The thick
horizontal deposits are called sills whereas the thinner ones are called sheets.
• Pair 3 is not correct: Batholiths - A large body of magmatic material that cools in the deeper depth of the crust
develops in the form of large domes. These are granitic bodies. Batholiths are the cooled portion of magma chambers.
Hence, option A is correct.

2nd Floor, 45 Pusa Road, Opp. Metro Pillar 128, Karol Bagh, New Delhi-110005
Ph: 08045248491, 07041021151 | Email: students@levelupias.com
Q89. Consider the following statements with respect to the types of Volcanoes:
1. Shield volcanoes are made up of basalt and low viscous lava.
2. Composite volcanoes erupt cooler and more viscous lavas than the basalt.
3. Caldera volcanoes are the most explosive volcanoes of the earth.
How many of the above statements is/are correct?
A. Only one
B. Only two
C. All three
D. None
Answer: C
Explanation:
• Statement 1 is correct: Shield Volcanoes are the largest of all the volcanoes on the earth. The Hawaiian volcanoes
are the most famous example. These volcanoes are mostly made up of basalt, a type of lava that is very fluid when
erupted. For this reason, these volcanoes are not erupted. These are formed by lava flows of low viscosity. These
volcanoes are not steep. They become explosive if somehow water gets into the vent; otherwise, they are less
explosive.
• Statement 2 is correct: Composite volcanoes are characterized by eruption of cooler and most viscous lavas than
the basalt. These volcanoes often result in the explosive eruptions. Along with lava, large quantities of pyroclastic
materials and ashes find their way to the ground. They are usually found at the destructive plate margins. They have
steep sides along with composite layer.
• Statement 3 is correct: Caldera volcanoes are the most explosive of the earth surface effect the surrounding
environment. They are usually so explosive that when they erupt, they tend to collapse on themselves rather than
building any structure. After the eruption of magma has ceased, the crater frequently turns into a lake at a later time.
This lake is called a ‘caldera’. Examples: Lonar in Maharashtra and Krakatao in Indonesia.
Hence, option C is correct.

Q90. Consider the following statements:


1. This state is the largest provider of natural rubber and black pepper to the Indian markets.
2. The average landholdings size of this state is lowest in India.
The above statements best referred to which of the following Indian state?
A. Kerala
B. Karnataka

2nd Floor, 45 Pusa Road, Opp. Metro Pillar 128, Karol Bagh, New Delhi-110005
Ph: 08045248491, 07041021151 | Email: students@levelupias.com
C. Tamil Nadu
D. Andhra Pradesh
Answer: A
Explanation:
• Kerala is known as the "Land of Spices" and account for almost 91-97% of India's black pepper production. It is also
a leading producer of natural rubber having 70% of India's natural rubber production.
• The average size of landholdings in Kerala is the lowest in India (0.12 ha).
Hence, option A is correct.

Q91. Which of the following statement is NOT correct with reference to Mangrove vegetation?
A. This vegetation mainly grows at tropical and subtropical latitudes.
B. Oviparous mode of reproduction is very common in mangrove species.
C. Leaves of many mangrove species have salt secreting glands.
D. In India, they are found along Andaman and Nicobar Island.
Answer: B
Explanation:
• Option (A) is correct: A mangrove is a small tree or shrub that flourishes in coastal areas with saline or brackish
water. These forests predominantly thrive in tropical and subtropical regions near the equator due to their
intolerance to freezing temperatures.
• Option (B) is not correct: The viviparous mode of reproduction is prevalent among mangrove species. In this
reproductive method, seeds germinate and transform into seedlings while remaining attached to the parent tree.
These seedlings, often referred to as propagules, engage in photosynthesis while still connected to the mother tree.
The parent tree provides essential water and nutrients to support the growth process. These buoyant propagules float
in the water for a certain period before eventually rooting themselves in suitable soil.
• Option (C) is correct: Mangroves, classified as halophytes, are resilient trees capable of thriving in challenging
coastal environments with high salinity. These trees have evolved intricate salt filtration mechanisms, including
leaves with salt-secreting glands, and develop complex root systems to withstand immersion in saltwater and the
effects of wave action.
• Option (D) is correct: The five prominent mangrove forests in India include the Sundarbans in West Bengal,
Pichavaram Mangroves in Tamil Nadu, Godavari–Krishna Mangroves in Andhra Pradesh, Bhitarkanika Mangroves in
Odisha, and Baratang Island Mangroves in the Andaman and Nicobar Islands.
Hence, option B is correct.

Q92. Which of the following factors can cause tsunamis in the ocean?
1. Undersea earthquakes
2. Volcanic eruption
3. Large meteorite strikes
4. Underwater landslide
Select the correct answer using the code given below:
A. 1 and 3 only
B. 2, 3 and 4 only
C. 1, 2 and 4 only
D. 1, 2, 3 and 4
Answer: D
Explanation:
2nd Floor, 45 Pusa Road, Opp. Metro Pillar 128, Karol Bagh, New Delhi-110005
Ph: 08045248491, 07041021151 | Email: students@levelupias.com
• Statement 1 is correct: Undersea earthquakes - These are the most common cause of tsunamis. When earthquake
faults rupture beneath the ocean, they can displace a large volume of water, creating a powerful wave that travels
outward.
• Statement 2 is correct: Volcanic eruptions - If a volcano erupts underwater, the explosion and ejected material can
displace water and generate a tsunami.
• Statement 3 is correct: Large meteorite strikes - If a large meteorite strikes the ocean, the impact can create a
massive crater and displace a huge amount of water, triggering a tsunami.
• Statement 4 is correct: Underwater landslides - These can occur along continental margins or near volcanic islands.
When a large mass of sediment or rock slides into the ocean, it can displace water and generate a tsunami.
Hence, option D is correct.

Q93. Which of the following statements clearly explains “Breaks in the monsoon”?
A. The date at which monsoon rainfall ends.
B. It denotes the phenomenon when monsoon winds collide with the mountains of the Western Ghats and the Himalayas.
C. It is the period in the South-West Monsoon during which rain fails to occur for days or weeks.
D. It occurs when there is an excess of rainfall, leading to flooding in the affected regions.
Answer: C
Explanation:
Breaks in the monsoon:
• Breaks in the monsoon refer to periods during the South-West Monsoon season when there is a temporary cessation
or reduction in rainfall over a region for several days or even weeks.
• These breaks are characterized by a lack of precipitation, which can have significant implications for agriculture, water
availability, and the overall economy of the affected areas.
• The occurrence of breaks in the monsoon can disrupt the normal progress of the monsoon season and lead to
variability in rainfall patterns, impacting farmers and communities dependent on agriculture for their livelihoods.
Hence, option C is correct.

Q94. Consider the following statements with reference to Coal:


1. It is the world’s most abundantly used fuel source.
2. Carbon concentration in the coal seams increases with age of coal.
3. United States has the world's largest proven coal reserves.
How many of the above statements is/are correct?
A. Only one
B. Only two
C. All three
D. None
Answer: C
Explanation:
• Statement 1 is correct: Coal has been one of the most abundantly used fuel sources globally for centuries. It played
a crucial role in the industrial revolution and has been a significant source of energy for power generation, industrial
processes, and heating.
• Statement 2 is correct: Coal is formed from plant material that has been subjected to high pressure and heat over
millions of years. As coal ages, the organic material undergoes chemical changes, leading to an increase in the
concentration of carbon in the coal seams

2nd Floor, 45 Pusa Road, Opp. Metro Pillar 128, Karol Bagh, New Delhi-110005
Ph: 08045248491, 07041021151 | Email: students@levelupias.com
• Statement 3 is correct: The United States is the country with the world’s largest proven coal reserves followed by
Russia, Australia, China and India.
Hence, option C is correct.

Q95. Consider the following statements with reference to ‘Tropical Evergreen Forest’:
1. This region has luxuriant vegetation including trees, shrubs and creepers.
2. Trees of this forest shed their leaves for about six to eight weeks in dry summer.
3. Ebony, mahogany, rosewood and cinchona are commonly found in this type of forest.
How many of the above statements is/are correct?
A. Only one
B. Only two
C. All three
D. None
Answer: B
Explanation:
• Statement 1 is correct: Tropical Evergreen Forests are characterized by dense and luxuriant vegetation. They typically
have a variety of plant life, including tall trees forming a canopy, shrubs, and various types of climbing plants or
creepers.
• Statement 2 is not correct: Tropical Evergreen Forests are characterized by the presence of trees that retain their
leaves throughout the year. These forests do not have a specific period of leaf shedding, and they maintain a
continuous green canopy.
• Statement 3 is correct: Ebony, mahogany, and rosewood are common tree species found in tropical evergreen
forests. These forests are known for their diverse flora, and these tree species contribute to the rich biodiversity of
the ecosystem.
Hence, option B is correct.

Q96. ‘Calicos’ and’ Chintz’ refers to which of the following?


A. Varieties of coffee produced in north-eastern part of the country.
B. Species of butterfly, that recently became state butterfly of Tamil Nadu.
C. State turmeric of Telangana that recently got GI tag.
D. Fine variety of cotton cloth of the country.
Answer: D
Explanation:
India was famous worldwide for the production of muslin, a very fine variety of cotton cloth, calicos, chintz and other
different varieties of fine cotton cloth.
• Calico: Calico is a plain-woven cotton fabric that is typically unbleached and unfinished. It is often printed with a
simple, small-scale pattern. Historically, calico was originally made in the Indian subcontinent and was later imported
to Europe.
• Chintz: Chintz is a glazed cotton fabric, usually printed with brightly coloured floral patterns. It often has a shiny finish
due to a glaze applied to the surface. Chintz gained popularity in Europe, especially during the 17th and 18th centuries.
Hence, option D is correct.

Q97. Consider the following statements with reference to ‘Atmosphere’:


1. The thickness of troposphere is maximum at the Equator.
2. The temperature decreases constantly in all the layers of atmosphere.
2nd Floor, 45 Pusa Road, Opp. Metro Pillar 128, Karol Bagh, New Delhi-110005
Ph: 08045248491, 07041021151 | Email: students@levelupias.com
3. Ozone is found in the stratosphere layer of the atmosphere.
How many of the above statements is/are NOT correct?
A. Only one
B. Only two
C. All three
D. None
Answer: A
Explanation:
• Statement 1 is correct: The troposphere is the lowermost layer of the atmosphere. Its average height is 13 km and
extends roughly to a height of 8 km near the poles and about 18 km at the equator. Thickness of the troposphere is
greatest at the equator because heat is transported to great heights by strong convectional currents. This layer
contains dust particles and water vapour. All changes in climate and weather take place in this layer.
• Statement 2 is not correct: The temperature decreases only in certain layers of the atmosphere such as in
Troposphere, Mesosphere but in certain layers such as Stratosphere, Thermosphere, etc. it is increasing.
• Statement 3 is correct: The stratosphere is found above the tropopause and extends up to a height of 50 km. One
important feature of the stratosphere is that it contains the ozone layer. This layer absorbs ultra-violet radiation
and shields life on the earth from intense, harmful form of energy.
Hence, option A is correct.

Q98. Consider the following crops:


1. Wheat
2. Barley
3. Jowar
4. Cotton
5. Mustard
Which of the above given crops comes under the category of ‘Rabi Crops’?
A. 1 and 4 only
B. 1, 2 and 5 only
C. 2 and 3 only
D. 1 and 5 only
Answer: B
Explanation:
• Rabi crops are sown in winter from October to December and harvested in summer from April to June. Some of the
important rabi crops are wheat, barley, peas, gram and mustard.
• Kharif crops are grown with the onset of monsoon in different parts of the country and these are harvested in
September-October. Important crops grown during this season are paddy, maize, jowar, bajra, tur (arhar), moong,
urad, cotton, jute, groundnut and soyabean.
Hence, option B is correct.

Q99. Which of the following statements best describe the term ‘Truck Farming’?
A. Plantation of ornamental flowers for commercial purpose.
B. Plantation of medicinal plant in a truck especially in hilly areas.
C. Growing one or more vegetable crops on a large scale for shipment to distant markets.
D. Plantation and transportation of shrub from landscape to mountainous areas.
Answer: C
2nd Floor, 45 Pusa Road, Opp. Metro Pillar 128, Karol Bagh, New Delhi-110005
Ph: 08045248491, 07041021151 | Email: students@levelupias.com
Explanation:
Truck Farming:
• Truck farming refers to the practice of growing one or more vegetable crops on a large scale with the intention of
shipping the produce to distant markets.
• It is a specialised horticulture practice where farmers are specialized in growing vegetables only. They produce on a
large scale for shipment to distant markets. The distance of truck farms from the market is governed by the distance
that a truck can cover overnight. hence the name.
Hence, option C is correct.

Q100. Equatorial regions have great potential of timber resources but its commercial exploitation is still under developed due
to :
1. Same species of trees do not occur in homogeneous stands.
2. There is no frozen surface to facilitate logging.
3. Tropical hardwood found here is too heavy to float in the river.
How many of the above statements is/are NOT the correct reasons?
A. Only one
B. Only two
C. All three
D. None
Answer: D
Explanation:
• Statement 1 is correct: Equatorial regions, known for their high biodiversity, often have various species of trees
growing together. This differs from temperate regions, where forests may consist of large stands of a single species,
making logging and processing more straightforward. In equatorial regions, the presence of multiple species in a single
area complicates logging operations, as different trees require different harvesting and processing techniques.
• Statement 2 is correct: In some colder regions, frozen surfaces, such as rivers or lakes, can provide a natural pathway
for transporting logs. During colder seasons, these surfaces freeze, allowing for the movement of heavy timber across
them. However, in equatorial regions, where temperatures are consistently warm, there are no frozen surfaces to
facilitate this kind of transportation. This lack of natural pathways makes it more challenging to transport timber,
increasing the costs and complexity of logging operations.
• Statement 3 is correct: Many tropical hardwood species found in equatorial regions are dense and heavy. This
characteristic makes them sink in water, which is a significant obstacle for transporting logs via rivers or waterways.
Unlike lighter woods that can float, these heavy hardwoods require alternative methods of transportation, such as
trucks or specialized equipment, adding to the cost and difficulty of commercial exploitation.
Hence, option D is correct.

2nd Floor, 45 Pusa Road, Opp. Metro Pillar 128, Karol Bagh, New Delhi-110005
Ph: 08045248491, 07041021151 | Email: students@levelupias.com

You might also like